HESI Set 02

Pataasin ang iyong marka sa homework at exams ngayon gamit ang Quizwiz!

How can a nurse minimize agitation in a disturbed client?

By limiting unnecessary interactions with the client

A client has a platelet count of 49,000/mL. The nurse should instruct the client to avoid which activity? 1 Ambulation 2 Blowing the nose 3 Visiting with children 4 The semi-Fowler's position

Correct2 Blowing the nose Patients with thrombocytopenia are at a greater risk of excessive bleeding in response to minimal trauma. The nurse should instruct the patient to avoid blowing their nose as this activity can increase the risk of bleeding. The following activities are not contraindicated with thrombocytopenia: ambulation, visiting with children, and semi-Fowler's position.

Which domain of the Nursing Interventions Classification taxonomy includes care that supports homeostatic regulation?

Domain 2

The nurse is caring for a client who requires an intravenous infusion. The nurse explains the reason for the procedure while assembling the kit for the infusion. What is the role of the nurse in this situation?

Educator

A client with multiple injuries from a motor vehicle accident now is permitted out of bed to a chair but is not permitted to bear weight on the lower extremities. When using a mechanical lift to transfer the client, it is essential that the nurse do what?

Fold the client's arms across the chest

A client's wound is healing. Which event occurs in the proliferative phase of wound healing?

Formation of "granulation" tissue

The school nurse conducts a class in nutrition planning for parents. What is the goal of school health nursing programs?

Health promotion

An obese adult develops an abscess after abdominal surgery. The wound is healing by secondary intention and requires repacking and redressing every four hours. Which diet should the nurse expect the health care provider to prescribe to best meet this client's immediate nutritional needs? 1. Low in fat and vitamin D 2. High in calories and fiber 3. Low in residue and bland 4. High in protein and vitamin C

High in protein and vitamin C

Which step in the research process is similar to the assessment step of the nursing process?

Identifying the problem

Which internal variable influences health beliefs and practices?

Intellectual background

A nurse finds that there is an inaccurate match between clinical cues and the nursing diagnosis. What is the category of the diagnostic error?

Interpreting

Which opposing conflict does a young adult face according to Erikson's theory of psychosocial development?

Intimacy versus Isolation

A nurse changing the dressing on the client's perineum would fall into which zone?

Intimate zone

Which practice would be suitable in the prevention of a pressure ulcer?

Keeping the client's skin directly off plastic surfaces

A nurse is explaining the nursing process to a nursing assistant. Which step of the nursing process should include interpretation of data collected about the client? 1 Evaluation 2 Data Collection 3 Nursing interventions 4 Proposed nursing care

Evaluation

What is the nurse assessing when checking the cardinal positions from the image?

Extraocular muscle function

The Magnet Recognition Program for health care organizations is based on fourteen forces of magnetism related to five magnet model components. Which force of magnetism is assessed to review the structural empowerment of the organization?

Personnel policies and programs

A client has a "prayer cloth" pinned to the hospital gown. The cloth is soiled from being touched frequently. What should the nurse do when changing the client's gown? 1 Make a new prayer cloth. 2 Discard the soiled prayer cloth. 3 Pin the prayer cloth to the clean gown. 4 Wash the prayer cloth with a detergent.

Pin the prayer cloth to the clean gown The prayer cloth has religious significance for the client and should be preserved as is. Making a new prayer cloth disregards what the prayer cloth means to the client. The prayer cloth is the property of the client and should not be discarded. Washing the prayer cloth with a detergent disregards what the prayer cloth means to the client; this never should be done without the client's permission.

Which statement is true about prescriptive theories?

Prescriptive theories are action-oriented.

A client has sensorineural hearing loss. Which finding in the client's history will alert the nurse to the most likely cause of the sensorineural hearing loss?

Prolonged exposure to noise

What is the mechanism of action for wet-to-damp saline-moistened gauze for wound debridement?

Removing the necrotic tissue mechanically

A block nurse is caring for an elderly couple in the neighborhood. What kind of service does block nursing offer to the elderly clients?

Running errands

Which of these is true about SOAP progress note method?

SOAP progress note includes assessment information.

A nurse is caring for a client with pulmonary tuberculosis. What must the nurse determine before discontinuing airborne precautions?

Sputum is free of acid-fast bacteria

The nurse finds the respiratory rate is 8 breaths per minute in a client who is on intravenous morphine sulfate. What should the nurse do immediately in this situation?

Stop administering the medication

While caring for a client receiving blood transfusion care, the nurse notices that the client is having an acute hemolytic reaction. What is the priority nursing intervention in this situation?

Stop the blood transfusion immediately.

Which statement made by a nursing student about Swanson's theory of caring needs correction?

Swanson's theory of caring provides a basis to help nurses understand how clients cope with uncertainty and the illness response

A client who had a total hip replacement is receiving continuous regional analgesia. The nurse recognizes what as the benefit of this treatment over conventional methods?

Systemic side effects are minimal

An older adult client who complains of difficulty breathing after a surgery is found to have decreased vital capacity on spirometry. Which nursing intervention should be performed in this situation?

Teach coughing and deep-breathing exercises.

The nurse is assessing four infants. Which infant does the nurse anticipate to be of abnormal weight?

The average birth weight of a newborn is 3.2 to 3.4 kg. An infant usually doubles his or her birth weight at 4 to 5 months of age. Therefore, infant 2's weight of 8.5 kg at 5 months is abnormal. Infant 1, weighing 6.1 kg, is of a normal weight. An infant has usually tripled his or her birth weight by around 1 year. Therefore, infants 3 and 4 are experiencing normal weight gain.

A client with dementia is admitted with a fractured hip after a fall at home. The client's family member witnessed the fall. Four hours after admission, the client's blood pressure increases to a moderately severe hypertensive level. The client pulls on the bedclothes continuously. The client's family member asks for pain medication for the client. What does the nurse concludes?

The client may be in pain and unable to respond appropriately.

The nurse is assessing a client who had knee replacement surgery. Which assessment finding gathered by the nurse is an example of subjective data?

The client's pain is 7 on a scale of 1 to 10

A client tells a nurse "I work in a factory that manufactures explosives." According to Maslow's hierarchy of needs, to which level of need does the given scenario refer?

The given scenario relates to the second level that includes safety and security needs

A client says "I feel frustrated because I do not spend enough time with my partner because of my job." According to Maslow's hierarchy of needs, which level of need does the given scenario refer?

The given scenario relates to the third level that includes love and belonging needs.

The nurse should monitor for which involuntary physiologic response in a client who is experiencing pain?

perspiring

Which are extrinsic factors responsible for falls in older adults? Select all that apply. 1 Impaired vision 2 Cognitive impairment 3 Environmental hazards 4 Inappropriate footwear 5 Improper use of assistive devices

3, 4, 5

Which age is considered the phallic stage according to Sigmund Freud's developmental theory?

3-6 years

A client is being admitted to a medical unit with a diagnosis of pulmonary tuberculosis. The nurse should assign the client to which type of room? 1 Private room 2 Semiprivate room 3 Room with windows that can be opened 4 Negative-airflow room

4

A client presents to the healthcare facility with abdominal pain. Which question should the nurse ask the client to obtain information about concomitant symptoms? 1 "Can you describe the pain?" 2 "Where exactly do you feel the pain?" 3 "Which activities make the pain worse?" 4 "What other discomfort do you experience?"

4

A nurse is assessing a client's degree of edema and finds 8 mm of depth. How does the nurse document this condition? 1 1+ 2 2+ 3 3+ 4 4+

4

An older adult with chills arrived to hospital. The nurse assesses the client's vital signs and determined the client has a fever. What would be the client's rectal temperature? 1 36.0ºC 2 36.8ºC 3 37.2ºC 4 38.5ºC

4

The nurse at a community healthcare center focuses on providing primary preventive care. What is the focus of primary preventive care? 1 Rehabilitating the client 2 Treating early stages of disease 3 Preventing complications from illness 4 Promoting health in healthy individuals

4

Which concept refers to respecting the rights of others? 1 Maturity 2 Systematicity 3 Inquisitiveness 4 Open-mindedness

4

Which physical assessment technique involves listening to the sounds of the body? 1 Palpation 2 Inspection 3 Percussion 4 Auscultation

4

An 80-year-old female is admitted to the hospital because of complications associated with severe dehydration. The client's daughter asks the nurse how her mother could have become dehydrated because she is alert and able to care for herself. The nurse's best response is: 1 "The body's fluid needs decrease with age because of tissue changes." 2 "Access to fluid may be insufficient to meet the daily needs of the older adult." 3 "Memory declines with age, and the older adult may forget to ingest adequate amounts of fluid." 4 "The thirst reflex diminishes with age, and therefore the recognition of the need for fluid is decreased.

4 "The thirst reflex diminishes with age, and therefore the recognition of the need for fluid is decreased.

The nurse plans care for a client with a somatoform disorder based on the understanding that the disorder is: 1 A physiological response to stress. 2 A conscious defense against anxiety. 3 An intentional attempt to gain attention. 4 An unconscious means of reducing stress

4 An unconscious means of reducing stress

What should the nurse do initially when obtaining consent for surgery? 1 Describe the risks involved in the surgery. 2 Explain that obtaining the signature is routine for any surgery. 3 Witness the client's signature, which the nurse's signature will document. 4 Determine whether the client's knowledge level is sufficient to give consent

4 Determine whether the client's knowledge level is sufficient to give consent

What should the nurse include in dietary teaching for a client with a colostomy? 1 Liquids should be limited to 1 L per day. 2 Non-digestible fiber and fruits should be eliminated. 3 A formed stool is an indicator of constipation. 4 The diet should be adjusted to include foods that result in manageable stools

4 The diet should be adjusted to include foods that result in manageable stools

Which site should be monitored for a pulse to assess the status of circulation to the foot? Select all that apply. 1 Carotid artery 2 Femoral artery 3 Popliteal artery 4 Dorsalis pedis artery 5 Posterior tibial artery

4,5

A registered nurse is teaching a nursing student about Erikson's theory of psychosocial development. To which age group does Industry versus Inferiority apply?

6- 11 years

A registered nurse is teaching a nursing student about Piaget's theory of cognitive development that includes four periods, which are related to age. Which age group corresponds with concrete operations?

7 to 11

The nurse informs a client's family that the client is in pain and does not wish to proceed with chemotherapy. What is the role of the nurse in this situation?

Advocate

A doctor asks a nurse to collect the medical history of a client. What nursing process should the nurse undertake?

Assessment

The client is receiving high-flow intravenous (IV) fluid replacement therapy. Which nursing assessment findings are consistent with fluid volume overload?

Bounding pulse, presence of dependent edema, and neck vein distention in the upright position

A client is diagnosed with AIDS. When examining the client's oral cavity, the nurse assesses white patchy plaques on the mucosa. The nurse recognizes that this finding most likely represents what opportunistic infection? 1 Cytomegalovirus 2 Histoplasmosis 3 Candida albicans 4 Human papillomavirus

Candida albicans White patchy plaques on the oral mucosa would most likely be a result of C. albicans, a yeastlike fungal infection. This condition is also known as "thrush."

An older client with macular degeneration comes to the eye clinic. Which response reported by the client does the nurse identify as consistent with the diagnosis?

Cannot see objects in the center of the visual field

The nurse assesses an elderly client with a diagnosis of dehydration and recognizes which finding as an early sign of dehydration?

Change in mental status

A child who reports shortness of breath, wheezing, and coughing is found to have pulmonary edema and is prescribed furosemide. Which nursing interventions would be beneficial to the client? Select all that apply.

Checking the child's weight every day Calculating the dose of drug as carefully as possible Assessing the child regularly to help prevent electrolyte loss

A registered nurse is teaching a nursing student about the concepts that make up a theory. Which point noted by the nursing student needs correction?

Concepts consist of interrelated theories.

The nurse explains a certain procedure to a client. The client seems shy, so the nurse encourages the client to ask questions. What is the correct term for this example of critical thinking by the nurse?

Confidence

A client is admitted with severe diarrhea that resulted in hypokalemia. The nurse should monitor for what clinical manifestations of the electrolyte deficiency? (Select all that apply.) 1 Diplopia 2 Skin rash 3 Leg cramps 4 Tachycardia 5 Muscle weakness

Correct 4 Tachycardia Correct 5 Muscle weakness

Which statement accurately describes correlational research?

Correlational research explores the relationships among variables of interest without any active intervention by the researcher.

A client reports diminished sensations of pain, touch, and temperature on the skin. The nurse touches the skin and finds it cool. Which skin changes should the nurse relate to the client's findings?

Decreased blood flow to the skin

What is the definition of descriptive research?

Descriptive research measures the characteristics of persons, situations, or groups.

Which instruction from the nurse to an 80-year-old client with thinning of a subcutaneous layer would be most beneficial?

Dress warmly in cold weather

A client comes to the medical clinic complaining of headaches. The nurse measures the blood pressure at 172/114. What should the nurse do first? 1 Page the on-call health care provider and continue to monitor the blood pressure. 2 Administer ibuprofen and have the client rest quietly for 20 minutes. 3 Elevate the head of the bed, provide reassurance, and reassess the blood pressure. 4 Place the client in the supine position, administer oxygen, and notify the health care provider.

Elevate the head of the bed, provide reassurance, and reassess the blood pressure. Blood pressure increases with pain and stress; reevaluation is critical before determining if the health care provider should be notified. Assessment should be completed before notifying the health care provider. Prescribing medications is a dependent function of the nurse, and medication should not be administered until the cause of the headache is determined. Oxygen is not indicated. The head of the bed should be elevated. The health care provider should be notified if a second blood pressure reading remains elevated.

Which statement about Henderson's theory of nursing care is correct?

Henderson organized the theory into 14 basic needs of the whole person and includes phenomena from the following domains of the client: physiological, psychological, sociocultural, spiritual, and developmental.

A nurse is advising a parent to teach the child impulse control and cooperative behaviors to avoid the risks of altered growth and development. In which stage does the nurse teach the parent about this, according to Erikson's theory of psychosocial development?

Initiative versus Guilt

Which action of the nurse would be most important to convey interest in starting a conversation with a client who has hearing loss?

Making eye contact with the client

The nurse assesses a client for orthostatic hypotension. The results are: Lying heart rate = 70 beats/minute, BP = 110/70; Sitting heart rate = 78 beats/minute, BP = 106/66; Standing heart rate = 85 beats/minute, BP = 100/64. The nurse would expect which prescription from the primary healthcare provider?

No prescription change

According to Benner et al., an expert nurse passes through five levels of proficiency when acquiring specialized nursing skills. What is the correct order of levels from lowest to highest?

Novice Advanced beginner Competent Proficient Expert

Which healthcare system is required after a physical or mental illness, injury, or chemical addiction?

Rehabilitation

A nurse is monitoring a client who is receiving an intravenous (IV) infusion of normal saline. What is a serious complication of IV therapy?

Shortness of breath with crackles

A client comes to the clinic complaining of a productive cough with copious yellow sputum, fever, and chills for the past 2 days. What is the first thing the nurse should do when caring for this client?

Take temperature

A client's serum potassium level has increased to 5.8 mEq/L (5.8 mmol/L). What action should the nurse implement first?

Take vital signs and notify the healthcare provider.

Which statement is applicable to Watson's theory of transpersonal caring?

Watson's theory defines the outcome of nursing activity in relation to the humanistic aspects of life

The registered nurse is teaching a nursing student about the skills to build a helping relationship with the client. Arrange the events of the helping relationship in chronological order. Correct 1. Planning enough time for the initial interaction Correct 2. Assessing the client's health status Correct 3. Providing information needed to understand and change behavior Correct 4. Achieving a smooth transition for the client to other caregivers as needed

perfect

What is the sequence of techniques used while assessing the abdomen? 1. Inspection 2. Auscultation 3. Percussion 4. Palpation

perfect

While assessing an older adult with decreased perception of touch, the nurse provides instructions to the client to reduce the risk associated with falling. Which statements made by the nurse are beneficial to the client? Select all that apply.

- look where your feet are placed while walking - wear shoes that give good support - if you are unable to change your positions frequently request assistance

A client with a history of hypothyroidism reports giddiness, excessive thirst, and nausea. Which parameter assessed by the nurse confirms the diagnosis as heat stroke? 1 Increased heart rate 2 Increased blood pressure 3 Decreased respiratory rate 4 Increased circulatory damage

1

A nurse is assessing a child who is accompanied by a parent. The parent has remarried and has another child from the second marriage. What kind of a family does this child belong to? 1 Blended family 2 Extended family 3 Alternative family 4 Single-parent family

1

A nurse notices cyanosis in a client with heart disease. Which site would the nurse assess to confirm cyanosis? 1 Lips 2 Sclera 3 Conjunctiva 4 Mucus membrane

1

An older adult with a history of diabetes reports giddiness, excessive thirst, and nausea. During an assessment, the nurse notices the client's body temperature as 105° F. Which condition does the nurse suspect in the client? 1 Heat stroke 2 Heat exhaustion 3 Accidental hypothermia 4 Malignant hyperthermia

1

In which situation does the nurse consider the family as context? 1 The nurse is caring for an individual with tonsillitis. 2 The nurse is caring for a dying client and all the family members. 3 The nurse is teaching young parents about caring for their toddler. 4 The nurse is assessing the needs of the family caregivers of a client.

1

What is the inflammation of the skin at the base of the nail called? 1 Paronychia 2 Koilonychia 3 Beau's lines 4 Splinter hemorrhage

1

Which approach is a comforting approach that communicates concern and support? 1 Touch 2 Listening 3 Knowing the client 4 Providing a positive presence

1

Which client's need should be considered high priority? 1 A client with dysphagia who is choking while eating 2 A client who needs discharge teaching about medications 3 A client who needs a dressing change of the surgical wound 4 A client who has a knowledge deficit regarding the use of an insulin pen

1

Which intervention reflects the nurse's approach of "family as a context"? 1 Trying to meet the client's comfort 2 Evaluating the client family's coping skills 3 Evaluating the client family's energy level 4 Trying to meet the client family's nutritional needs

1

Which nurse collaborates directly with the client to establish and implement a basic plan of care after admission? 1 Primary nurse 2 Nurse clinician 3 Nurse coordinator 4 Clinical nurse specialist

1

A client with hypothermia is brought to the emergency department. What treatment does the nurse anticipate when the patient is in the emergency department? 1 Core rewarming with warm fluids 2 Ambulation to increase metabolism 3 Frequent oral temperature assessments 4 Gastric tube feedings to increase fluid volume

1 Core rewarming with warm fluids

Which degree of edema will result in a 6-mm deep indentation upon pressure application? 1 4+ 2 3+ 3 2+ 4 1+

2

Which factor can elevate the oxygen saturation during an assessment? 1 Nail polishes 2 Carbon monoxide 3 Intravascular dyes 4 Skin pigmentation

2

A nurse overhears an unlicensed assistive personnel (UAP) talking with a client about the client's marital and family problems. The nurse identifies that the UAP is providing false reassurance when the UAP states: 1 "I agree; I think you should get a divorce." 2 "Everything will be fine, just wait and see." 3 "You should be glad that you have such a loving family." 4 "In the scheme of things, you do not have a major problem."

2 "Everything will be fine, just wait and see."

A hospitalized client experiences a fall after climbing over the bed's side rails. Upon reviewing the client's medical record, the nurse discovers that restraints had been prescribed but were not in place at the time of the fall. What information should the nurse include in the follow-up incident report? 1 A statement that the nursing staff was not at fault because the client initiated the accident. 2 A listing of facts related to the incident as witnessed by the nurse. 3 The name of the nurse who was responsible for implementing the restraints. 4 The potential reasons why the restraints were not in place at the time of the fall.

2 A listing of facts related to the incident as witnessed by the nurse.

A client expresses concern about being exposed to radiation therapy because it can cause cancer. What should the nurse emphasize when informing the client about exposure to radiation? 1 The dosage is kept at a minimum. 2 Only a small part of the body is irradiated. 3 The client's physical condition is not a risk factor. 4 Nutritional environment of the affected cells is a risk factor.

2 Only a small part of the body is irradiated.

It is appropriate for the nurse to pull up on the client's skin, release it, and determine if the skin returns immediately to its original position to assess for: 1 Pain tolerance 2 Skin turgor 3 Ecchymosis formation 4 Tissue mass

2 Skin turgor

A client who is HIV positive is admitted to a surgical unit after an orthopedic procedure. The nurse should institute appropriate precautions with the awareness that HIV is highly transmissible through: (Select all that apply.) 1 feces. 2 blood. 3 semen. 4 urine. 5 sweat. 6 tears

2 blood. 3 semen

A client who is human immunodeficiency virus (HIV) positive is admitted to a surgical unit after an orthopedic procedure. The nurse should institute appropriate precautions with the awareness that HIV is highly transmissible through what means? Select all that apply. 1 Feces 2 Blood 3 Semen 4 Urine 5 Sweat 6 Tears

2,3

An 89-year-old client with osteoporosis is admitted to the hospital with a compression fracture of the spine. The nurse identifies that a factor of special concern when caring for this client is the client's: 1 Irritability in response to deprivation 2 Decreased ability to recall recent facts 3 Inability to maintain an optimal level of functioning 4 Gradual memory loss resulting from change in environment

3 Inability to maintain an optimal level of functioning

Nursing actions for the older adult should include health education and promotion of self-care. Which is most important when working with the older adult client? 1 Encouraging frequent naps 2 Strengthening the concept of ageism 3 Reinforcing the client's strengths and promoting reminiscing 4 Teaching the client to increase calories and focusing on a high carbohydrate diet

3 Reinforcing the client's strengths and promoting reminiscing

A client becomes hostile when learning that amputation of a gangrenous toe is being considered. After the client's outburst, what is the best indication that the nurse-client interaction has been therapeutic? 1 Increased physical activity 2 Absence of further outbursts 3 Relaxation of tensed muscles 4 Denial of the need for further discussion

3 Relaxation of tensed muscles Relaxation of muscles and facial expression are examples of nonverbal behavior; nonverbal behavior is an excellent index of feelings because it is less likely to be consciously controlled. Increased activity may be an expression of anger or hostility. Clients may suppress verbal outbursts despite feelings and become withdrawn. Refusing to talk may be a sign that the client is just not ready to discuss feelings.

When teaching about aging, the nurse explains that older adults usually have: 1 Inflexible attitudes 2 Periods of confusion 3 Slower reaction times 4 Some senile dementia

3 Slower reaction times

What clinical indicators should the nurse expect a client with hyperkalemia to exhibit? Select all that apply.

- diarrhea - weakness - dysrhythmias

Which term refers to a blowing sound created by turbulence caused by narrowing of arteries while assessing for carotid pulse? 1 Bruit 2 Ectropion 3 Entropion 4 Borborygmi

1

Which sites would be safe and inexpensive for temperature measurement? Select all that apply. 1 Skin 2 Oral 3 Axilla 4 Rectal 5 Tympanic membrane

1,3

Which of the following statements about a case manager is correct?

"A case manager has the ability to establish an appropriate care plan based on the assessment of clients and families."

A nurse is recalling Piaget's theory of cognitive development. Which statement is a characteristic of the concrete operations stage?

"A child is able describe a process without actually doing it."

A nurse is recalling Piaget's theory of cognitive development. What is the characteristic of the preoperational stage?

"An infant may learn to think with the use of symbols and mental images."

A hospitalized client is on contact precautions for methicillin-resistant Staphylococcus aureus (MRSA). Which statement by an unlicensed assistive personnel (UAP) (Canada: continuing care assistant, CCA) indicates a need for further teaching?

"I will remove the gown, then the gloves, before washing my hands.

What instruction would the nurse be most likely to give a client with reduced sensory perception to prevent injury from scalding?

"Use a bath thermometer."

Which intrinsic factors may contribute to falls in older adults? Select all that apply.

- deconditioning - impaired vision

While assessing a client with chills and fever, the nurse observes that the febrile episodes are followed by normal temperatures and that the episodes are longer than 24 hours. Which fever pattern does the nurse anticipate? 1 Relapsing 2 Sustained 3 Remittent 4 Intermittent

1

While assessing a client's vascular system, the nurse finds that pulse strength is diminished or barely palpable. Which documentation is appropriate in this situation? 1 1+ 2 2+ 3 3+ 4 4+

1

When caring for a client who is receiving enteral feedings, the nurse should take which measure to prevent aspiration? 1 Elevate HOB 30-45 degrees. 2 Decrease flow rate at night. 3 Check for residual daily. 4 Irrigate regularly with warm tap water.

1 Elevate HOB 30-45 degrees.

A client with a leg prosthesis and a history of syncopal episodes is being admitted to the hospital. When formulating the plan of care for this client, the nurse should include that the client is at risk for: 1 Falls 2 Impaired cognition 3 Imbalanced nutrition 4 Impaired gas exchange

1 Falls The client is at risk for falls related to the leg prosthesis and history of syncope. There is no evidence or contributing factors in the patient scenario of the other nursing problems.

A nurse applies an ice pack to a client's leg for 20 minutes. The cold application will cause what physiological effect? 1 Local anesthesia 2 Peripheral vasodilation 3 Depression of vital signs 4 Decreased viscosity of blood

1 Local anesthesia Cold reduces the sensitivity of pain receptors in the skin. In addition, local blood vessels constrict, limiting the amount of edema and its related pressure and discomfort. Local blood vessels constrict. Local cold applications do not depress vital signs. Local cold applications do not affect blood viscosity directly.

A nurse is assisting a client to transfer from the bed to a chair. What should the nurse do to widen the client's base of support during the transfer? 1 Spread the client's feet away from each other. 2 Move the client on the count of three. 3 Instruct the client to flex the muscles of the internal girdle. 4 Stand close to the client when assisting with the move

1 Spread the client's feet away from each other. Spreading the feet apart widens the base of support. A wide base of support lowers the center of gravity, thereby increasing stability

When assessing a client's blood pressure, the nurse notes that the blood pressure reading in the right arm is 10 mm Hg higher than the blood pressure reading in the left arm. The nurse understands that this finding: 1 is a normal occurrence. 2 may indicate atherosclerosis. 3 can be attributed to aortic disease. 4 indicates lymphedema

1 is a normal occurrence.

According to Erikson's theory, at which age would a child develop self-control and independence?

18 months to 3 years old

A 78-year-old client who has hypertension is beginning treatment with furosemide. Considering the client's age, what should the nurse teach the client to do? 1 Limit fluids at bedtime. 2 Change positions slowly. 3 Take the medication between meals. 4 Assess the skin for breakdown daily.

2

Which term refers to the exaggeration of the posterior curvature of the thoracic spine? 1 Lordosis 2 Scoliosis 3 Kyphosis 4 Osteoporosis

3

While performing a physical assessment of a female client, a nurse notices hair on the client's upper lip, chin, and cheeks. Which condition may result in this condition? 1 Aging 2 Poor nutrition 3 Endocrine disease 4 Arterial insufficiency

3

A nurse speaking in support of the best interest of a vulnerable client reflects the nurse's duty of: 1 Caring. 2 Veracity. 3 Advocacy. 4 Confidentiality

3 Advocacy. The nurse has a professional duty to advocate for a client by promoting what is best for the client. This is accomplished by ensuring that the client's needs are met and by protecting the client's rights.

The nurse receives a report on a newly admitted client who is positive for Clostridium difficile. Which category of isolation would the nurse implement for this client? 1 Airborne precautions 2 Droplet precautions 3 Contact precautions 4 Protective environment

3 Contact precautions Contact precautions should be used for direct client or environmental contact with blood or body fluids from an infected client. This includes colonization of infection with multidrug-resistant organisms (MDRO) such as MRSA, stool infected with Clostridium difficle, draining wounds where secretions are not contained, or scabies.

A hospital has threatened to refuse the discharge of a newborn until the parents pay part of the hospital bill. The nurse is aware that the legal term that best describes this situation is: 1 False threats 2 Assault and battery 3 False imprisonment 4 Breach of confidentiality

3 False imprisonment

A nurse receives abnormal results of diagnostic testing. What action should the nurse take first? 1 Inform the client of the results. 2 Ensure that the results are placed in the client's medical record. 3 Notify the client's healthcare provider of the results. 4 Request the test be re-done to ensure accuracy.

3 Notify the client's healthcare provider of the results.

A client has been instructed to stop smoking. The nurse discovers a pack of cigarettes in the client's bathrobe. What is the nurse's initial action? 1 Notify the health care provider. 2 Report this to the nurse manager. 3 Tell the client that the cigarettes were found. 4 Discard the cigarettes without commenting to the client

3 Tell the client that the cigarettes were found.

The nurse assesses a client's pulse and documents the strength of the pulse as 3+. The nurse understands that this indicates the pulse is: 1 diminished. 2 normal. 3 full. 4 bounding.

3 full. The strength of a pulse is a measurement of the force at which blood is ejected against the arterial wall. A 3+ rating indicates a full increased pulse. A zero rating indicates an absent pulse. A rating of a 1+ indicates a diminished pulse that is barely palpable. A 2+ rating is an expected/normal pulse, and a 4+ rating is a bounding pulse.

While assessing a neonate's temperature, the nurse observes a drop in the body temperature. What is the most appropriate reason for this temperature drop? 1 Increased basal metabolic rate 2 Decreased involuntary shivering 3 Increased voluntary movements 4 Decreased nonshivering thermogenesis

4

While assessing the client's skin, a nurse notices a skin condition, the pathophysiology of which involves increased visibility of oxyhemoglobin caused by an increased blood flow due to capillary dilation. Which condition is associated with this client? 1 Pallor 2 Vitiligo 3 Cyanosis 4 Erythema

4

A visitor from a room adjacent to a client asks the nurse what disease the client has. The nurse responds, "I cannot discuss any client's illness with you." What legal issue supports the nurse's response? 1 Libel 2 Slander 3 Negligence 4 Invasion of privacy

4 Invasion of privacy

The nurse interviews a client about a current health problem. The nurse then obtains and documents the client's temperature, blood pressure, and heart rate. Which step of the nursing process is involved in this situation?

Assessment

The registered nurse (RN) provides nursing care based on the critical thinking model known as the nursing process. What is the correct order of steps in the nursing process?

Assessment Diagnosis Planning Implementation Evaluation

Normal, harsh, hollow, tubular, blowing sounds heard over the trachea and larynx.

Bronchial breath sounds

A nurse is teaching staff members about the legal terminology used in child abuse. What definition of battery should the nurse include in the teaching? A. Maligning a person's character while threatening to do bodily harm B. A legal wrong committed by one person against property of another C. The application of force to another person without lawful justification D. Behaving in a way that a reasonable person with the same education would not

C. The application of force to another person without lawful justification. Battery means touching in an offensive manner or actually injuring another person. Battery refers to actual bodily harm rather than threats of physical or psychological harm. Battery refers to harm against persons, not property. Behaving in a way that a reasonable person with the same education would not is the definition of negligence.

Which clients should be considered for assessing the carotid pulse?

Client with cardiac arrest Client under physiologic shock

A client is ambivalent about making a change in health behavior. Which stage of health behavior does the nurse suspect?

Contemplation

A client is in a state of ambivalence. Which of these stages of health behavior will the nurse suspect?

Contemplation

What type of research explores the interrelationship among variables of interest without any active intervention by a researcher?

Correlation research

A client who experienced extensive burns is receiving intravenous fluids to replace fluid loss. The nurse should monitor for which initial sign of fluid overload?

Crackles in the lungs

A registered nurse is educating a nursing student about descriptive theories. Which point stated by the nursing student needs correction?

Descriptive theories help direct specific nursing activities

Which is an example of indirect contact transmission of microorganisms?

Dirty hands. Indirect contact transmission involves the transfer of microorganisms from a source to a host by passive transfer from a contaminated article or hands

Which of these databases should the nurse use to obtain a broad view on biomedical and pharmaceutical studies?

EMBASE

In which process of Swanson's theory is the nurse engaging when explaining neonatal care to a parent?

Enabling

What is considered to be the highest priority for an assault victim who presents to the emergency department?

Ensuring the client's emotional and physical safety

Which are extrinsic factors responsible for falls in older adults?

Environmental hazards Inappropriate footwear Improper use of assistive devices

Which first line medication would the nurse state is used to treat anaphylactic reactions?

Epinephrin

What is the difference between evidence-based practice and quality improvement?

Evidence-based practice focuses on the implementation of evidence already known into practice

What information is a nurse likely to find in the clinical implications section of an article?

Explanation regarding the method of applying findings in a practice setting

Which opposing conflict would a middle-aged adult face according to Erikson's theory of psychosocial development?

Generativity versus Self-Absorption and Stagnation

Which is an example of an actual nursing diagnosis?

Impaired social interaction

Which step in the nursing process would involve promoting a safe environment for the client?

Implementation

Which statement about Orem's theory needs to be corrected?

It describes factors supporting the health of the family

Which statement about Orem's theory needs to be corrected?

It describes factors supporting the health of the family.

What is exploratory research?

It is a study designed to develop a hypothesis about the relationships among phenomena

Which statement defines "information" gathered by the nurse?

It is the organization and interpretation of data.

A nurse has made a nursing diagnosis without validating the data obtained from the client. Into what category does this error fall?

Labeling

While caring for a group of clients from different ethnicities, the nurse observes that a client from Ireland is stoic and not complaining about pain. Which theory should the nurse follow in this situation?

Leininger's Theory

A 70-year-old client needs to undergo heart surgery but cannot afford it. The client seeks the assistance of a nurse. Which is the preferred program that the nurse may suggest?

Medicare

According to Sigmund Freud's developmental theory, which developmental age is called the latent stage?

Middle Childhood

The nurse is caring for clients in the pulmonary unit and suspects that one has tuberculosis. What is the priority nursing intervention in this situation?

Move patient into airborne isolation room

Which are the core roles for an advanced practice registered nurse (APRN)? Select all that apply.

Nurse practitioner Clinical nurse specialist (CNS) Certified nurse midwife (CNM) Certified registered nurse anesthetist (CRNA)

Which diseases can be transmitted from client to client by droplet infection?

Pertussis Diphtheria

Which serum laboratory values in a client with urinary problems may indicate the risk of developing muscle weakness and cardiac arrhythmias?

Potassium of 7.02 mEq/L (7.02 mmol/L)

A client with bone cancer is receiving hospice care at home. The hospice program also provides respite care. What is the purpose of respite care?

Providing short-term relief to the family caregiver

While assessing a client with chills and fever, the nurse observes that the febrile episodes are followed by normal temperatures and that the episodes are longer than 24 hours. Which fever pattern does the nurse anticipate?

Relapsing

The primary healthcare provider confirms that the client has myopia. Which type of test did the nurse perform to help the primary healthcare provider reach this conclusion?

Snellen eye chart

A nurse is caring for a client diagnosed with methicillin-resistant Staphylococcus aureus (MRSA) in the urine. The healthcare provider orders an indwelling urinary catheter to be inserted. Which precaution should the nurse take during this procedure?

Surgical asepsis

Which theory provides a basis for identifying and testing nursing care behaviors to determine if caring improves patient health outcomes?

Swanson's theory of caring

Which system is used by a health care facility to determine certain aspects of client satisfaction?

The Hospital Consumer of Assessment of Healthcare Providers and Systems (HCAHPS)

A nursing student under the supervision of a registered nurse is performing a pulse assessment. While preparing to assess the client, the registered nurse asks the nursing student to check the apical pulse after assessing the radial pulse. What could be the reason behind for this change?

The client may have a dysrhythmia

Which feature according to Benner is observed in a nurse at the "proficient" level?

The nurse focuses on managing care rather than managing skills.

Which characteristic indicates that nursing is a profession?

The nurse is required to follow a code of ethics.

Which example indicates that the nurse is following evidence-based practice?

The nurse reads current nursing journals and uses the latest scientific methods

Which description is associated with a hematoma?

The visible swelling due to extravasation of blood of sufficient size

A client with colon cancer is receiving hospice care at home. What is the focus of hospice care?

To ease the pain from illness

During an assessment, the client complains of tenderness when the nurse palpates the calf muscle. What would be the nurse's next assessment?

To evaluate for swelling, warmth and muscle firmness

What is the main focus of community health nursing?

To improve the quality of health in a population

What is the goal of nursing according to Watson's theory?

To promote health, restore the client to health, and prevent illness

What is the purpose of block and parish nursing?

To provide services to older clients

What is the function of the Professional Standards Review Organizations (PSROs) set up by the federal government?

To review the quality, quantity, and cost of hospital care

A client's sputum smears for acid-fast bacilli (AFB) are positive, and transmission-based airborne precautions are prescribed. What should the nurse teach visitors to do?

Wear a particulate respirator mask.

A client has a stage III pressure ulcer. Which nursing intervention can prevent further injury by eliminating shearing force? 1 Maintain the head of the bed at 35 degrees or less. 2 With the help of another staff member, use a drawsheet when lifting the client in bed. 3 Reposition the client at least every 2 hours and support the client with pillows. 4 At least once every 8 hours, perform passive range-of-motion exercises of all extremities.

With the help of another staff member, use a drawsheet when lifting the client in bed.

The nurse is caring for a client who is in pain following surgery. The nurse informs the primary health care provider about the client's request for pain medication. What is the role of the nurse in this situation?

advocate

A nurse is caring for a client with diarrhea. The nurse anticipates a decrease in which clinical indicator?

tissue turgor

A client with a known history of opioid addiction is treated for multiple stab wounds to the abdomen. After surgical repair the nurse notes that the client's pain is not relieved by the prescribed morphine injections. The nurse realizes that the failure to achieve pain relief indicates that the client is probably experiencing what phenomenon?

tolerance

While assessing the pupils of a client, a healthcare professional notices pupillary dilatation. Which drug intake might have resulted in this condition? 1 Heroin 2 Atropine 3 Morphine 4 Pilocarpine

2

A client who underwent a physical examination reports itching after 2 days. Which condition should the nurse suspect? 1 Eczema 2 Hypersensitivity 3 Contact dermatitis 4 Anaphylactic shock

3

Which physical skin finding indicates opioid abuse? 1 Diaphoresis 2 Red, dry skin 3 Needle marks 4 Spider angiomas

3

Which type of breathing pattern alteration is manifested with hypercarbia? 1 Eupnea 2 Tachypnea 3 Hypoventilation 4 Kussmaul's respiration

3

The registered nurse (RN) is delegating tasks to several health care team members caring for a client with a bacterial infection and high fever. Which statements by the registered nurse would be most appropriate for the licensed practical nurse (LPN)?

"You should administer an antipyretic after the meal." "You should administer prescribed antibiotics to the client twice a day." Correct 5 "You should collect data that can be used in the assessment of client's condition.

The nurse is caring for a client admitted with fluid overload. Which tasks are most appropriate to be delegated to the patient care associate? Select all that apply.

- documenting vital signs - documenting urine output - repositioning the client every one or two hours

The nurse is communicating with an older adult who has a hearing disability. Which intervention by the nurse is beneficial to promote communication? Select all that apply.

- giving the client a chance to speak - making sure that the client knows you are speaking - keeping the communication concise

Which gerontologic assessment findings of the auditory system are related to the inner ear? Select all that apply.

- hair cell degeneration - reduced blood supply to the cochlea - less effective vestibular apparatus in the semicircular canals

A registered nurse is teaching a nursing student about how to communicate with a client who is cognitively impaired. Which statements made by the nursing student indicate a need for further education? Select all that apply.

- i should use visual cues - i should speak in a normal tone of voice - i should face the client so that he or she can see my mouth

A client who underwent surgery feels pain in the lower abdomen. The nurse provides pain relief but the client is still reporting pain. Which actions of the nurse would help the client to get relief? Select all that apply.

- looking for different distraction technique - involving the client's family in creating a new plan for pain relief

A new nurse has been assigned to a school-aged child who is in contact isolation for methicillin-resistant Staphylococcus aureus (MRSA). The primary nurse observes the new nurse during morning care. Which behavior should the primary nurse address to improve isolation technique?

-While changing the bed the nurse wears gloves but no gown Organisms spread by contact could be present on linen and furniture. Because the nurse will likely come in contact with the bed or the sheets, a gown should be worn. A mask is not required for contact isolation. Clean gloves are appropriate; sterile gloves are not necessary. No other protective gear is needed, because the nurse's hands are the only part of the body that might touch the child. Equipment brought into the room must be disinfected before it is removed from the room.

A client with a history of severe diarrhea for the past 3 days is admitted for dehydration. The nurse anticipates that which intravenous (IV) solution will be prescribed initially?

0.9% sodium chloride

A client who has been battling cancer of the ovary for 7 years is admitted to the hospital in a debilitated state. The healthcare provider tells the client that she is too frail for surgery or further chemotherapy. When making rounds during the night, the nurse enters the client's room and finds her crying. Which is the most appropriate intervention by the nurse? 1 Sit down quietly next to the bed and allow her to cry. 2 Pull the curtain and leave the room to provide privacy for the client. 3 Explain to the client that her feelings are expected and they will pass with time. 4 Observe the length of time the client cries and document her difficulty accepting her impending death.

1

A nurse in a long-term health care setting will introduce a client who has a PhD to the other clients. The client tells the nurse, "I wish to be called Doctor." How should the nurse respond? 1 "Your wish will be respected." 2 "Why do you want to be called Doctor?" 3 "Residents here call one another by their first names." 4 "Wouldn't it be better if the others do not know you are a doctor?"

1

A nurse is teaching a client about proper hair hygiene and how to protect his or her hair from lice. Which statement made by the client indicates ineffective learning? 1 "I will soak the comb used to remove lice for 15 minutes in cold water." 2 "I will remove any detectable nits by using a metal nit comb after shampooing." 3 "I will shampoo thoroughly with pediculicide in cold water at a basin or sink." 4 "I will seal nonwashable items in plastic bags if unable to dry clean or vacuum."

1

A nurse is teaching continuing care assistants about ways to prevent the spread of infection. It would be appropriate for the nurse to emphasize that the cycle of the infectious process must be broken, which is accomplished primarily through what? 1 Hand washing before and after providing client care 2 Cleaning all equipment with an approved disinfectant after use 3 Wearing personal protective equipment (PPE) when providing client care 4 Using medical and surgical aseptic techniques at all times

1

A nurse is teaching members of a health care team how to help disabled clients stand and transfer from the bed to a chair. To protect the caregivers from injury, the nurse teaches them to lift the client by first placing their arms under the client's axillae and doing what next? 1 Bending and then straightening their knees 2 Bending at the waist and then straightening the back 3 Placing one foot in front of the other and then leaning back 4 Placing pressure against the client's axillae and then raising their arms

1

A nurse on the medical-surgical unit tells other staff members, "That client can just wait for the lorazepam; I get so annoyed when people drink too much." What does this nurse's comment reflect? 1 Demonstration of a personal bias 2 Problem solving based on assessment 3 Determination of client acuity to set priorities 4 Consideration of the complexity of client care

1

A student nurse is assessing the blood pressure of a client with the client's arm unsupported. What are the expected errors in the obtained readings? 1 False high reading 2 False low diastolic reading 3 False high systolic reading 4 False high diastolic readin

1

After assessing the muscle functionality of a client, the nurse assigns a grade of F (fair) on the Lovett scale in the client. What is the muscle functionality of the client? 1 Full range of motion with gravity 2 Full range of motion with gravity eliminated 3 Full range of motion against gravity with full resistance 4 Full range of motion against gravity with some resistance

1

Nurses care for clients in a variety of age groups. In which age group is the occurrence of chronic illness the greatest? 1 Older adults 2 Adolescents 3 Young children 4 Middle-aged adults

1

On the second day of hospitalization a client is discussing with the nurse concerns about unhealthy family relationships. During the nurse-client interaction the client begins to talk about a job problem. The nurse's response is, "Let's go back to what we were just talking about." What therapeutic communication technique did the nurse use? 1 Focusing 2 Restating 3 Exploring 4 Accepting

1

The nurse is discussing discharge plans with a client who had a myocardial infarction. The client states, "I'm worried about going home." The nurse responds, "Tell me more about this." What interviewing technique did the nurse use? 1 Exploring 2 Reflecting 3 Refocusing 4 Acknowledging

1

Which feature is characteristic of a risk nursing diagnosis? 1 The diagnosis does not have related factors. 2 The diagnosis can be used in any health state. 3 The defining characteristics support the diagnostic judgment. 4 The defining characteristics are supported by a client's readiness.

1

Which pulse site is used for the Allen's test? 1 Ulnar 2 Radial 3 Brachial 4 Femoral

1

Which response by the nurse during a client interview is an example of back channeling? 1 "All right, go on..." 2 "What else is bothering you?" 3 "Tell me what brought you here." 4 "How would you rate your pain on a scale of 0 to 10?"

1

Which step of the nursing process is directly affected if the nurse does not make a nursing diagnosis? 1 Planning 2 Evaluation 3 Assessment 4 Implementation

1

A nurse anticipates that a hospitalized client will be transferred to a nursing home. When should the nurse begin preparing the client for the transfer? 1 At the time of admission 2 After a relative gives permission 3 When the client talks about future plans 4 As soon as the client's transfer has been approved

1 At the time of admission Preparation of clients for discharge to their own home or to a nursing home should begin on the day of admission. The client gives permission for transfer to a nursing home

A nurse is teaching members of a health care team how to help disabled clients stand and transfer from the bed to a chair. To protect the caregivers from injury, the nurse teaches them to lift the client by first placing their arms under the client's axillae and next: 1 Bending and then straightening their knees 2 Bending at the waist and then straightening the back 3 Placing one foot in front of the other and then leaning back 4 Placing pressure against the client's axillae and then raising their arms

1 Bending and then straightening their knees The leg bones and muscles are used for weight bearing and are the strongest in the body. Using the knees for leverage while lifting the client shifts the stress of the transfer to the caregiver's legs. By using the strong muscles of the legs the back is protected from injury.

A nurse is caring for a newly admitted client in a long-term care facility. The nurse notes that the client has a decreased attention span and cannot concentrate. The nurse suspects which effects of sensory deprivation? 1 Cognitive response 2 Emotional response 3 Perceptual response 4 Physical response

1 Cognitive response

A 2-g sodium diet is prescribed for a client with stage 2 hypertension, and the nurse teaches the client the rationale for this diet. The client reports distaste for the food. The primary nurse hears the client request that the family "bring in a ham and cheese sandwich and fries." What is the most effective nursing intervention? 1 Discuss the diet with the client and family. 2 Tell the client why salty foods should not be eaten. 3 Explain the dietary restriction to the client's visitors. 4 Ask the dietitian to teach the client and family about sodium restrictions

1 Discuss the diet with the client and family.

While instructing a community group regarding risk factors for coronary artery disease, the nurse provides a list of risk factors that cannot be modified. What should be included on the list? 1 Heredity 2 Hypertension 3 Cigarette smoking 4 Diabetes mellitus

1 Heredity Heredity refers to genetic makeup and cannot be changed. Cigarette smoking is a lifestyle habit that involves behavior modification. Hypertension and diabetes mellitus are risk factors of coronary artery disease that can be controlled with diet, medication, and exercise.

A nurse is preparing to change a client's dressing. What is the reason for using surgical asepsis during this procedure? 1 Keeps the area free of microorganisms. 2 Confines microorganisms to the surgical site. 3 Protects self from microorganisms in the wound. 4 Reduces the risk for growing opportunistic microorganisms.

1 Keeps the area free of microorganisms. Surgical asepsis means that practices are employed to keep a defined site or objects free of all microorganisms. Confining microorganisms to the surgical site and protecting self from microorganisms in the wound applies to personal protective equipment and medical asepsis. Reducing the risk for growing opportunistic microorganisms applies to medical asepsis.

Nurses care for clients in a variety of age groups. In which age group is the occurrence of chronic illness the greatest? 1. Older adults 2. Adolescents 3. Young children 4. Middle-aged adults

1 Older adults

A nurse is teaching a client how to use the call bell/call light system. Which level of Maslow's Hierarchy of Needs does this nursing action address? 1 Safety 2 Self-esteem 3 Physiological 4 Interpersonal

1 Safety

A registered nurse is teaching a nursing student about systems theories with a specific reference to Neuman's systems theory. Which statements made by the nursing student post teaching are accurate? Select all that apply.

"Factors that change the environment also affect an open system." "The components are interrelated and share a common purpose to form a whole." "An open system interacts with the environment, with an exchange of information between the system and the environment."

A teenager begins to cry while talking with the nurse about the problem of not being able to make friends. What is the most therapeutic nursing intervention? 1 Sitting quietly with the client. 2 Telling the client that crying is not helpful. 3 Suggesting that the client play a board game. 4 Recommending how the client can change this situation

1 Sitting quietly with the client. Sitting quietly with the client conveys the message that the nurse cares and accepts the client's feelings; this helps to establish trust. Telling the client that crying is not helpful negates feelings and the client's right to cry when upset. Distraction (suggesting that the client play a board game) closes the door on further communication of feelings. After a trusting relationship has been established, the nurse can help the client explore the problem in more depth.

An emaciated older adult with dementia develops a large pressure ulcer after refusing to change position for extended periods of time. The family blames the nurses and threatens to sue. What is considered when determining the source of blame for the pressure ulcer? 1 The client should have been turned regularly. 2 Older clients frequently develop pressure ulcers. 3 The nurse is not responsible to the client's family. 4 Nurses should respect a client's right not to be moved.

1 The client should have been turned regularly. Clients should change position at least every two hours to prevent pressure ulcers. The nurse should not deviate from this standard of practice because of the cognitively-impaired client's refusal to move. The nurse was negligent for not changing the client's position. Although pressure ulcers may occur, nursing care must include preventive measures. The family is included in the health team. When a capable client refuses necessary health care, the nurse should provide health teaching to promote understanding of the treatment plan. If the client makes an informed decision after an explanation, then the client's rights must be respected; however, this client is cognitively impaired.

An older client who is receiving chemotherapy for cancer has severe nausea and vomiting and becomes dehydrated. The client is admitted to the hospital for rehydration therapy. Which interventions have specific gerontologic implications the nurse must consider? (Select all that apply.) 1 Assessment of skin turgor 2 Documentation of vital signs 3 Assessment of intake and output 4 Administration of antiemetic drugs 5 Replacement of fluid and electrolytes

1 Assessment of skin turgor 4 Administration of antiemetic drugs 5 Replacement of fluid and electrolytes

To decrease abdominal distention following a client's surgery, what actions should the nurse take? (Select all that apply.) 1 Encourage ambulation 2 Give sips of ginger ale 3 Auscultate bowel sounds 4 Provide a straw for drinking 5 Offer an opioid analgesic

1 Encourage ambulation 3 Auscultate bowel sounds Ambulation will stimulate peristalsis, increasing passage of flatus and decreasing distention. Monitoring bowel sounds is important because it provides information about peristalsis. Carbonated beverages, such as ginger ale, increase flatulence and should be avoided. Using a straw should be avoided because it causes swallowing of air, which increases flatulence. Opioids will slow peristalsis, contributing to increased distention.

A nurse assesses the vital signs of a 50-year-old female client and documents the results. Which of the following are considered within normal range for this client? (Select all that apply.) 1 Oral temperature 98.2° F 2 Apical pulse 88 beats per minute and regular 3 Respiratory rate of 30 per minute 4 Blood pressure 116/78 mm Hg while in a sitting position 5 Oxygen saturation of 92%

1 Oral temperature 98.2° F 2 Apical pulse 88 beats per minute and regular 4 Blood pressure 116/78 mm Hg while in a sitting position

Alternative therapy measures have become increasingly accepted within the past decade, especially in the relief of pain. Which methods qualify as alternative therapies for pain? (Select all that apply.) 1 Prayer 2 Hypnosis 3 Medication 4 Aromatherapy 5 Guided imagery

1 Prayer 2 Hypnosis 4 Aromatherapy 5 Guided imagery

The client asks the nurse to recommend foods that might be included in a diet for diverticular disease. Which foods would be appropriate to include in the teaching plan? (Select all that apply.) 1 Whole grains 2 Cooked fruit and vegetables 3 Nuts and seeds 4 Lean red meats 5 Milk and eggs

1 Whole grains 2 Cooked fruit and vegetables 5 Milk and eggs

Which workers would the nurse consider to be at high risk of developing dermatitis? Select all that apply. 1 Dry cleaners 2 Dye workers 3 Lathe operators 4 Hospital workers 5 Agricultural workers

1, 2

A client with a recent history of head trauma is at risk for orthostatic hypotension. Which assessment findings observed by the nurse would relate to this diagnosis? Select all that apply. 1 Fainting 2 Headache 3 Weakness 4 Lightheadedness 5 Shortness of breath

1,3,4

A nurse is assessing an older adult during a regular checkup. Which findings during the assessment are normal? Select all that apply. 1 Loss of turgor 2 Urinary incontinence 3 Decreased night vision 4 Decreased mobility of ribs 5 Increased sensitivity to odors

1,3,4

Which physiologic changes may occur during the first trimester of pregnancy? Select all that apply. 1 Fatigue 2 Increased libido 3 Morning sickness 4 Breast enlargement 5 Braxton Hicks contractions

1,3,4

What are the goals of care when working with families according to the family health system? Select all that apply. 1 To improve family health or well-being 2 To help the family prepare for later transitions 3 To assist in family management of illness conditions 4 To promote positive family behaviors to achieve essential tasks 5 To achieve health outcomes related to the family's areas of concern

1,3,5

What are physiologic symptoms assessed in a client with sleep deprivation? Select all that apply. 1 Ptosis and blurred vision 2 Agitation and hyperactivity 3 Confusion and disorientation 4 Increased sensitivity to pain 5 Decreased auditory alertness

1,5

What are the steps of evidence-based practice (EBP) in order?

1. Asking the relevant clinical question 2. Collecting the most relevant and best evidence 3. Critically appraising the evidence collected 4. Integrating all evidence with one's clinical expertise and client preferences 5. Evaluating the practice decision 6. Sharing the outcomes of EBP changes with ot

A client is admitted to the emergency department with dyspnea, a productive cough, and fever. The healthcare provider suspects pneumonia and writes prescriptions. Place the nursing actions in the order they should be performed.

1. Elevate the head of the bed. 2. Insert a catheter to establish venous access 3. Obtain a sputum culture. 4. Administer the prescribed intravenous piggyback antibiotic 5. Review the results of the sensitivity test.

A client is admitted with a sudden onset of dyspnea and chest pain. What are the interventions in the order in which the nurse will perform them to provide comfort to the client?

1. Notifying the Rapid Response Team 2. Reassuring the client and family members 3. elevate the head of the bed to help the client breathe easier 4. Prepare oxygen therapy and blood gas analysis 5. Monitoring and assessing for other changes

A client with cancer is informed that the chemotherapy is no longer working and that death is inevitable. Keeping in mind Kübler-Ross's stages of death and dying, place the following nursing interventions that are most appropriately associated with each stage in order from the stage of denial to acceptance. 1. Provide maximal comfort measures. 2. Avoid confronting the client. 3. Redirect negative feelings constructively. 4. Help the client identify realistic versus unrealistic goals. 5. Help the client celebrate the simple pleasures in everyday life.

1.Avoid confronting the client. 2.Redirect negative feelings constructively. 3.Help the client identify realistic versus unrealistic goals. 4.Help the client celebrate the simple pleasures in everyday life. 5.Provide maximal comfort measures. DABDA During the denial stage, the nurse needs to avoid confronting the client's behavior because denial at this early stage is a self-protective mechanism. It is unwise to confront a client's coping mechanism because it leaves the client unprotected. During the anger stage, the nurse needs to accept the client's behavior and redirect negative feelings constructively. During the bargaining stage, the client generally seeks to achieve a goal. The nurse needs to help the client be realistic in this endeavor. During the depression stage, the nurse should encourage the client to engage in simple pleasures, such as sitting in the sun. During the acceptance stage, the nurse should provide comfort measures based on needs and desires.

Place each step of the nursing process in the order that it should be used. a. Identify goals for care. b. Develop a plan of care. c. State client's nursing needs. d. Implement nursing interventions. e. Obtain client's nursing history.

1.Obtain client's nursing history. 2. State client's nursing needs. 3. Identify goals for care 4. Develop a plan of care. 5. Implement nursing interventions.

A client admitted to the hospital with a diagnosis of malabsorption syndrome exhibits signs of tetany. The nurse concludes that the tetany was precipitated by the inadequate absorption of which electrolyte? 1 Sodium 2 Calcium 3 Potassium 4 Phosphorus

2

A client complains of sudden muscle weakness during times of anger or laughter that may occur at any time during the day. Which condition should be suspected in this client? 1 Insomnia 2 Cataplexy 3 Narcolepsy 4 Sleep apnea

2

A client experiencing chills and fever is admitted to the hospital. After assessing the client's vitals and medical history, the nurse concluded that the client's fever pattern is remittent. Which assessment finding led to this conclusion? 1 The client's temperature returns to an acceptable value at least once in the past 24 hours 2 The client's fever spikes and falls without a return to normal temperature levels 3 Periods of febrile episodes and periods with acceptable temperature values occur 4 The client has a constant body temperature continuously above 38°C with minimal fluctuation

2

A client requests information about the prescribed medication regimen. What is the best response by the nurse? 1 Give a computer printout about the medication to the client. 2 Ask the client to state what is already known about the medication. 3 Advise talking to the primary healthcare provider to seek information about the medication. 4 Delegate the task of sharing information about the medication to the licensed practical nurse.

2

A client with an abdominal wound infected with methicillin-resistant Staphylococcus aureus (MRSA) is scheduled for a computed tomography (CT) scan of the abdomen. To ensure client and visitor safety during transport, the nurse should implement which precaution? 1 No special precautions are required. 2 Cover the infected site with a dressing. 3 Drape the client with a covering labeled biohazardous. 4 Place a surgical mask on the client.

2

A newly hired nurse during orientation is approached by a surveyor from the department of health. The surveyor asks the nurse about the best way to prevent the spread of infection. What is the most appropriate nursing response? 1 "Let me get my preceptor." 2 "Wash your hands before and after any client care." 3 "Clean all instruments and work surfaces with an approved disinfectant." 4 "Ensure proper disposal of all items contaminated with blood or body fluids."

2

A nurse is caring for a client admitted with cardiovascular disease. During the assessment of the client's lower extremities, the nurse notes that the client has thin, shiny skin; decreased hair growth; and thickened toenails. What might this indicate? 1 Venous insufficiency 2 Arterial insufficiency 3 Phlebitis 4 Lymphedema

2

A nurse is caring for a client on bed rest. How can the nurse help prevent a pulmonary embolus? 1 Limit the client's fluid intake. 2 Teach the client how to exercise the legs. 3 Encourage use of the incentive spirometer. 4 Maintain the knee gatch position at an angle.

2

A nurse is caring for a client who is having diarrhea. To prevent an adverse outcome, the nurse should most closely monitor what patient data or assessment finding? 1 Skin condition 2 Fluid and electrolyte balance 3 Food intake 4 Fluid intake and output

2

A nurse is reviewing how a hyperglycemic client's blood glucose can be lowered. The nurse recalls that the chemical that buffers the client's excessive acetoacetic acid is what? 1 Potassium 2 Sodium bicarbonate 3 Carbon dioxide 4 Sodium chloride

2

After changing a dressing that was used to cover a draining wound on a client with vancomycin-resistant enterococci (VRE), the nurse should take which step to ensure proper disposal of the soiled dressing? 1 Place the dressing in the bedside trash can. 2 Place the dressing in a red bag/hazardous materials bag. 3 Contact Environmental Services personnel to pick up the dressing. 4 Transport the dressing to the laboratory to be placed in the incinerator.

2

The nurse is educating a client about tips for speaking up to help the client to be more involved in his or her treatment. Which statement made by the client indicates the need for further education? 1 "I should pay attention to the care." 2 "I should make assumptions regarding the treatment." 3 "I should speak up and have questions or concerns." 4 "I should learn about medical tests that are prescribed."

2

The nurse is providing restraint education to a group of nursing students. The nurse should include that it is inappropriate to use a restraint device to do what? 1 Prevent a client from pulling out an intravenous (IV) when there is concern that the client cannot follow instructions or is confused. 2 Prevent an adult client from getting up at night when there is insufficient staffing on the unit. 3 Maintain immobilization of a client's leg to prevent dislodging a skin graft. 4 Keep an older adult client from falling out of bed following a surgical procedure.

2

Which activity by the community nurse can be considered an illness prevention strategy? 1 Encouraging the client to exercise daily 2 Arranging an immunization program for chicken pox 3 Teaching the community about stress management 4 Teaching the client about maintaining a nutritious diet

2

Which landmark is correct for a nurse to use when auscultating the mitral valve? 1 Left fifth intercostal space, midaxillary line 2 Left fifth intercostal space, midclavicular line 3 Left second intercostal space, sternal border 4 Left fifth intercostal space, sternal border

2

Which position is indicated to assess the musculoskeletal system and is contraindicated in clients with respiratory difficulties? 1 Sims position 2 Prone position 3 Supine position 4 Knee-chest position

2

Which professional standard does the nurse feel is most important for critical thinking? 1 Logical thinking 2 Evaluation criteria 3 Accurate knowledge 4 Relevant information

2

Which skill would most likely be associated with an effective nurse leader? 1 Recognizing his or her own limitations 2 Delegating work appropriately 3 Displaying confidence in his or her knowledge base 4 Respecting the rights, beliefs, wishes, and values of clients

2

Which stage of Piaget's theory of cognitive development does the nurse observe in a preschooler? 1 Sensorimotor 2 Preoperational 3 Formal operations 4 Concrete operations

2

Which statement best describes a diagnostic label? 1 It is a condition that responds to nursing interventions. 2 It describes the essence of the client's response to health conditions. 3 It describes the characteristics of the client's response to health conditions. 4 It is identified from the client's assessment data and associated with the diagnosis.

2

A newly hired nurse, during orientation, is approached by a surveyor from the department of health. The surveyor asks the nurse about the best way to prevent the spread of infection. What is the most appropriate nursing response? 1 "Let me get my preceptor." 2 "Wash your hands before and after any client care." 3 "Clean all instruments and work surfaces with an approved disinfectant." 4 "Ensure proper disposal of all items contaminated with blood or body fluids.

2 "Wash your hands before and after any client care."

A client who has reached the stage of acceptance in the grieving process appears peaceful, but demonstrates a lack of involvement with the environment. How should the nurse address this behavior? 1 Ignore the client's behavior when possible. 2 Accept the behavior the client is exhibiting. 3 Explore the reality of the situation with the client. 4 Encourage participation within the client's environment.

2 Accept the behavior the client is exhibiting. Detachment is a coping mechanism that the client needs, especially when faced with the inevitability of death; the nurse should accept this behavior.

A nurse is caring for a client admitted with cardiovascular disease. During the assessment of the client's lower extremities, the nurse notes that the client has thin, shiny skin, decreased hair growth, and thickened toenails. The nurse understands that this may indicate: 1 Venous insufficiency 2 Arterial Insufficiency 3 Phlebitis 4 Lymphedema

2 Arterial Insufficiency Clients suffering from arterial insufficiency present with pale colored extremities when elevated and dusky red colored extremities when lowered. Lower extremities may also be cool to touch, pulses may be absent or mild, and skin may be shiny, thin, with decreased hair growth, and thickened nails. Clients suffering from venous insufficiency often have normal colored extremities, normal temperature, normal pulses, marked edema, and brown pigmentation around ankles. Phlebitis is an inflammation of a vein that occurs most often after trauma to the vessel wall, infection, and immobilization. Lymphedema is swelling in one or more extremities that is a direct result from impaired flow of the lymphatic system.

Elbow restraints have been prescribed for a confused client to keep the client from pulling out a nasogastric tube and indwelling urinary retention catheter. What is most important for the nurse to do? 1 Have the prescription renewed every 48 hours 2 Assess the client's condition every hour 3 Provide range of motion to the client's elbows every shift 4 Document output from the tube and catheter every two hours

2 Assess the client's condition every hour A restraint impedes the movement of a client; therefore, a client's condition needs to be assessed every hour. All restraints are required to be represcribed every 24 hours. Restraints should be removed and activity and skin care provided at least every two hours to prevent contractures and skin breakdown. Output from tubes may be monitored hourly, but generally do not need to be documented as frequently as every two hours. Generally output from tubes is emptied, measured, and documented at the end of each shift. A client who is in critical condition or in the immediate postoperative period may have urinary output measured hourly because this reflects cardiovascular status.

A nurse who promotes freedom of choice for clients in decision-making best supports which principle? 1 Justice 2 Autonomy 3 Beneficence 4 Paternalism

2 Autonomy The principle of autonomy relates to the freedom of a person to form his or her own judgments and actions. The nurse promotes autonomy nonjudgmentally so as not to infringe on the decisions or actions of others

According to Kübler-Ross, during which stage of grieving are individuals with serious health problems most likely to seek other medical opinions? 1 Anger 2 Denial 3 Bargaining 4 Depression

2 Denial Denial includes feelings that the health care provider has made a mistake, so the client seeks additional opinions.

A client reports fatigue and dyspnea and appears pale. The nurse questions the client about medications currently being taken. In light of the symptoms, which medication causes the nurse to be most concerned? 1 Famotidine (Pepcid) 2 Methyldopa (Aldomet) 3 Ferrous sulfate (Feosol) 4 Levothyroxine (Synthroid)

2 Methyldopa (Aldomet) Methyldopa is associated with acquired hemolytic anemia and should be discontinued to prevent progression and complications. Famotidine will not cause these symptoms; it decreases gastric acid secretion, which will decrease the risk of gastrointestinal bleeding. Ferrous sulfate is an iron supplement to correct, not cause, symptoms of anemia. Levothyroxine is not associated with red blood cell destruction.

What is the most important factor relative to a therapeutic nurse-client relationship when a nurse is caring for a client who is terminally ill? 1 Knowledge of the grieving process 2 Personal feelings about terminal illness 3 Recognition of the family's ability to cope 4 Previous experience with terminally ill clients

2 Personal feelings about terminal illness To be effective in a relationship with a client, the nurse must know and understand personal feelings about terminal illness and death.

A nurse educator is presenting information about the nursing process to a class of nursing students. What definition of the nursing process should be included in the presentation? 1 Procedures used to implement client care. 2 Sequence of steps used to meet the client's needs. 3 Activities employed to identify a client's problem. 4 Mechanisms applied to determine nursing goals for the client.

2 Sequence of steps used to meet the client's needs.

A toddler screams and cries noisily after parental visits, disturbing all the other children. When the crying is particularly loud and prolonged, the nurse puts the crib in a separate room and closes the door. The toddler is left there until the crying ceases, a matter of 30 or 45 minutes. Legally, how should this behavior be interpreted? 1 Limits had to be set to control the child's crying. 2 The child had a right to remain in the room with the other children. 3 The child had to be removed because the other children needed to be considered. 4 Segregation of the child for more than half an hour was too long a period of time.

2 The child had a right to remain in the room with the other children. Legally, a person cannot be locked in a room (isolated) unless there is a threat of danger either to the self or to others. Limit setting in this situation is not warranted. This is a reaction to separation from the parent, which is common at this age. Crying, although irritating, will not harm the other children. A child should never be isolated.

A client with osteoporosis is encouraged to drink milk. The client refuses the milk, explaining that it causes gas and bloating. Which food should the nurse suggest that is rich in calcium and digested easily by clients who do not tolerate milk? 1 Eggs 2 Yogurt 3 Potatoes 4 Applesauce

2 Yogurt Yogurt, which contains calcium, is digested more easily because it contains the enzyme lactase, which breaks down milk sugar. Yogurt contains approximately 274 to 415 mg of calcium for an 8-oz container, depending on how it is prepared. Eggs contain approximately 22 mg of calcium. One potato contains approximately 7 to 20 mg of calcium, depending on how it is prepared. Eight ounces of applesauce contain approximately 3 mg of calcium.

The nurse expects a client with an elevated temperature to exhibit what indicators of pyrexia? (Select all that apply.) 1 Dyspnea 2 Flushed face 3 Chest pain 4 Increased pulse rate 5 Increased blood pressure

2 Flushed face 4 Increased pulse rate Increased body heat dilates blood vessels, causing a flushed face. The pulse rate increases to meet increased tissue demands for oxygen in the febrile state. Fever may not cause difficult breathing. Pain is not related to fever. Blood pressure is not expected to increase with fever.

The nurse recognizes that which are important components of a neurovascular assessment? (Select all that apply.) 1 2 Orientation 3 Capillary refill 4 Pupillary response 5 Respiratory rate 6 Pulse and skin temperature 7 Movement and sensation

2 Orientation 5 Respiratory rate 6 Pulse and skin temperature A neurovascular assessment involves evaluating of nerve and blood supply to an extremity involved in an injury. The area involved may include an orthopedic and/or soft tissue injury. A correct neurovascular assessment should include evaluating of capillary refill, pulses, warmth and paresthesias, and movement and sensation. Orientation, pupillary response, and respiratory rate are components of a neurological assessment.

What are the best ways for a nurse to be protected legally? (Select all that apply.) 1 Ensure that a therapeutic relationship with all clients has been established. 2 Provide care within the parameters of the state's nurse practice act. 3 Carry at least $100,000 worth of liability insurance. 4 Document consistently and objectively. 5 Clearly document a client's non-adherence to the medical regimen.

2 Provide care within the parameters of the state's nurse practice act. 4 Document consistently and objectively. 5 Clearly document a client's non-adherence to the medical regimen.

Which parts of the body assessed by the nurse would confirm a diagnosis of frostbite? Select all that apply. 1 Axilla 2 Fingers 3 Ear lobes 4 Forehead 5 Upper thorax

2,3

When should the nurse consider family members as the primary source of information? Select all that apply. 1 The client is an elderly adult. 2 The client is an infant or child. 3 The client is brought in as an emergency. 4 The client is critically ill and disoriented. 5 The client visits the outpatient department.

2,3,4

The nurse is gathering a client's health history. Which information does should the nurse classify as biographical information? Select all that apply. 1 Symptoms 2 Client's age 3 Family structure 4 Type of insurance 5 Occupation status

2,4,5

A 50-year-old client being seen for a routine physical asks why a stool specimen for occult blood testing has been prescribed when there is no history of health problems. What is an appropriate nursing response? 1 "You will need to ask your healthcare provider; it is not part of the usual tests for people your age." 2 "There must be concern of a family history of colon cancer; that is a primary reason for an occult blood stool test." 3 "It is performed routinely starting at your age as part of an assessment for colon cancer." 4 "There must have been a positive finding after a digital rectal examination performed by your healthcare provider."

3

A client is admitted with a suspected malignant melanoma on the arm. When performing the physical assessment, what would the nurse expect to find? 1 Large area of petechiae 2 Red birthmark that has recently become lighter in color 3 Brown or black mole with red, white, or blue areas 4 Patchy loss of skin pigmentation

3

A client is to receive a transfusion of packed red blood cells (PRBCs). The nurse should prepare for the transfusion by priming the blood IV tubing with which solution? 1 Lactated Ringer solution 2 5% dextrose and water 3 0.9% normal saline 4 0.45% normal saline

3

A client on hospice care is receiving palliative treatment. A palliative approach involves planning measures aimed to do what? 1 Restore the client's health. 2 Promote the client's recovery. 3 Relieve the client's discomfort. 4 Support the client's significant others.

3

A client tells the nurse, "I am so worried about the results of the biopsy they took today." The nurse overhears the nursing assistant reply, "Don't worry. I'm sure everything will come out all right." What does the nurse conclude about the nursing assistant's answer? 1 It shows empathy. 2 It uses distraction. 3 It gives false reassurance. 4 It makes a value judgment.

3

A client with coronary artery disease has a sudden episode of cyanosis and a change in respirations. The nurse starts oxygen administration immediately. Legally, should the nurse have administered the oxygen? 1 The oxygen had not been prescribed and therefore should not have been administered. 2 The symptoms were too vague for the nurse to determine a need for administering oxygen. 3 The nurse's observations were sufficient, and therefore oxygen should have been administered. 4 The primary healthcare provider should have been called for a prescription before the nurse administered the oxygen.

3

A client's breath has a sweet, fruity odor. Which condition is likely affecting this client? 1 Gum disease 2 Uremic acidosis 3 Diabetic acidosis 4 Infection inside a cast

3

A nurse assesses drainage on a surgical dressing and documents the findings. Which documentation is most informative? 1 "Moderate amount of drainage." 2 "No change in drainage since yesterday." 3 "A 10-mm-diameter area of drainage at 1900 hours." 4 "Drainage is doubled in size since last dressing change."

3

A nurse assesses for hypocalcemia in a postoperative client. What is one of the initial signs that might be present? 1 Headache 2 Pallor 3 Paresthesias 4 Blurred vision

3

A nurse assesses the lungs of a client and auscultates soft, crackling, bubbling breath sounds that are more obvious on inspiration. This assessment should be documented as what? 1 Vesicular 2 Bronchial 3 Crackles 4 Rhonchi

3

A nurse is caring for a client who underwent cardiac catheterization. The client's skin was found to be blanched, and there was formation of edema of 15.2 cm (1-6 inches) at the site of catheterization. Upon further assessment, the skin was found to be cool, and the client complains of tenderness. Which condition does the nurse expect? 1 Phlebitis 2 Infection 3 Infiltration 4 Circulatory overload

3

A nurse is palpating the peripheral pulse of different clients. Which client has an unacceptable heart rate? 1 Client 1 2 Client 2 3 Client 3 4 Client 4

3

A nurse is planning to provide self-care health information to several clients. Which client should the nurse anticipate will be most motivated to learn? 1 A 55-year-old client who had a mastectomy and is very anxious about her body image 2 An 18-year-old client who smokes cigarettes and is in denial about the dangers of smoking 3 A 56-year-old client who had a heart attack last week and is requesting information about exercise 4 A 47-year-old client who has a long-leg cast after sustaining a broken leg and is still experiencing severe pain

3

A nurse is preparing a community health program for senior citizens. The nurse teaches the group that what physical findings are typical in older adults? 1 A loss of skin elasticity and a decrease in libido 2 Impaired fat digestion and increased salivary secretions 3 Increased blood pressure and decreased hormone production 4 An increase in body warmth and some swallowing difficulties

3

A registered nurse instructs a nursing student to use knowledge and experience to choose proper strategies to use to care for clients. Which critical-thinking skill does the registered nurse refer to? 1 Analysis 2 Evaluation 3 Explanation 4 Interpretation

3

An adolescent who had an inguinal hernia repair is being prepared for discharge home. The nurse provides instructions about resumption of physical activities. Which statement by the adolescent indicates that the client understands the instructions? 1 "I can ride my bike in about a week." 2 "I don't have to go to gym class for 3 months." 3 "I can't perform any weightlifting for at least 6 weeks." 4 "I can never participate in football again."

3

How many levels of critical thinking exist for nursing judgment? Record your answer using a whole number.

3

Nursing actions for an older adult should include health education and promotion of self-care. Which is most important when working with an older adult client? 1 Encouraging frequent naps 2 Strengthening the concept of ageism 3 Reinforcing the client's strengths and promoting reminiscing 4 Teaching the client to increase calories and focusing on a high-carbohydrate diet

3

The nurse finds that the client's fever spikes and falls without a return to a normal level. Which pattern of fever is this a characteristic of? 1 Relapsing 2 Sustained 3 Remittent 4 Intermittent

3

The nurse is caring for a client admitted with chronic obstructive pulmonary disease (COPD). The nurse should monitor the results of which laboratory test to evaluate the client for hypoxia? 1 Red blood cell count 2 Sputum culture 3 Arterial blood gas 4 Total hemoglobin

3

The nurse is developing a nursing diagnosis for a client after surgery. The nurse documents the "related to" factor as first time surgery. Which assessment activity enabled the nurse to derive this conclusion? 1 The nurse notes nonverbal signs of discomfort. 2 The nurse observes the client's position in bed. 3 The nurse asks the client to explain the surgery. 4 The nurse asks the client to rate the severity of pain.

3

The nurse reviews a medical record and is concerned that the client may develop hyperkalemia. Which disease increases the risk of hyperkalemia? 1 Crohn disease 2 Cushing disease 3 End-stage renal disease 4 Gastroesophageal reflux disease

3

The registered nurse is teaching a nursing student about providing care to an older adult with dementia. Which statement made by the nursing student indicates a need for further education? 1 "I should serve food that is easy to eat." 2 "I should assist the client with eating." 3 "I should monitor weight and food intake once in a month." 4 "I should offer food supplements that are tasty and easy to swallow."

3

To prevent septic shock in the hospitalized client, what should the nurse do? 1 Maintain the client in a normothermic state. 2 Administer blood products to replace fluid losses. 3 Use aseptic technique during all invasive procedures. 4 Keep the critically ill client immobilized to reduce metabolic demands.

3

To prevent thrombophlebitis in the immediate postoperative period, which action is most important for a nurse to include in the client's plan of care? 1 Increase fluid intake. 2 Restrict fluids. 3 Encourage early mobility. 4 Elevate the knee gatch of the bed.

3

What is the appropriate blood pressure of a 12-year-old client? 1 95/65 mm Hg 2 105/65 mm Hg 3 110/65 mm Hg 4 119/75 mm Hg

3

What should the nurse teach the parents of an infant about the use of car seats? 1 The infant should ride in a front-facing car safety seat. 2 The infant should ride in a car safety seat until one year of age. 3 The infant should be restrained properly in a federally approved car safety seat. 4 The infant should always ride in a car seat restrained to the front seat of the car.

3

When caring for a client with venous insufficiency, the nurse would implement which nursing measure? 1 Apply abdominal girdle as needed. 2 Remove compression stockings for client ambulation. 3 Elevate the client's legs above heart level. 4 Keep the upper extremities elevated.

3

When meeting the unique preoperative teaching needs of an older adult, the nurse plans a teaching program based on which principle about learning? 1 It reduces general anxiety. 2 It is negatively affected by aging. 3 It requires continued reinforcement. 4 It necessitates readiness of the learner.

3

Which assessing technique involves tapping a client's skin with the fingertips to cause vibrations in the underlying tissues? 1 Palpation 2 Inspection 3 Percussion 4 Auscultation

3

Which assessment is expected when a client is placed in the lithotomy position during physical examination? 1 Assessment of the heart 2 Assessment of the rectum 3 Assessment of the female genitalia 4 Assessment of the musculoskeletal system

3

Which clinical condition will result in changes in the integrity of the arterial walls and small blood vessels? 1 Contusion 2 Thrombosis 3 Atherosclerosis 4 Tourniquet effect

3

Which component of decision-making refers to the duties and activities an individual is employed to perform? 1 Authority 2 Autonomy 3 Responsibility 4 Accountability

3

Which developmental changes should be evaluated in girls around 12 years of age? 1 Motor skills 2 Visual acuity 3 Skeletal growth 4 Hormonal changes

3

Which site is best used to inspect a client who is suspected to have jaundice? 1 Skin 2 Palm 3 Sclera 4 Conjunctiva

3

While auscultating the heart, a healthcare provider notices S3 heart sounds in four clients. Which client is at more risk for heart failure? 1 Child client 2 Pregnant client 3 Older adult client 4 Young adult client

3

While inspecting the external eye structure of a client, a nurse finds bulging of the eyes. Which condition can be suspected in the client? 1 Eye tumors 2 Hypothyroidism 3 Hyperthyroidism 4 Neuromuscular injury

3

A nurse hired to work in a metropolitan hospital provides services for a culturally diverse population. One of the nurses on the unit says it is the nurses' responsibility to discourage "these people" from bringing all that "home medicine stuff" to their family members. Which response by the recently hired nurse is most appropriate? 1 "Hospital policies should put a stop to this." 2 "Everyone should conform to the prevailing culture." 3 "Nontraditional approaches to health care can be beneficial." 4 "You are right because they may have a negative impact on people's health."

3 "Nontraditional approaches to health care can be beneficial." Studies demonstrate that some nontraditional therapies are effective. Culturally competent professionals should be knowledgeable about other cultures and beliefs. Many health care facilities are incorporating both Western and nontraditional therapies. The statement "Everyone should conform to the prevailing culture" does not value diversity. The statement "You are right because they may have a negative impact on people's health" is judgmental and prejudicial. Some cultural practices may bring comfort to the client and may be beneficial, and they may not interfere with traditional therapy.

A nurse discusses the philosophy of Alcoholics Anonymous (AA) with the client who has a history of alcoholism. What need must self-help groups such as AA meet to be successful? 1 Trust 2 Growth 3 Belonging 4 Independence

3 Belonging

What effect of povidone-iodine (Betadine) does a nurse consider when using it on the client's skin before obtaining a specimen for a blood culture? 1 Makes the skin more supple 2 Avoids drying the skin as does alcohol 3 Eliminates surface bacteria that may contaminate the culture 4 Provides a cooling agent to diminish the feeling from the puncture wound

3 Eliminates surface bacteria that may contaminate the culture Povidone-iodine exerts bactericidal action that helps eliminate surface bacteria that will contaminate culture results

An older client is apprehensive about being hospitalized. The nurse realizes that one of the stresses of hospitalization is the unfamiliarity of the environment and activity. How can the nurse best limit the client's stress? 1 Use the client's first name. 2 Visit with the client frequently. 3 Explain what the client can expect. 4 Listen to what the client has to say.

3 Explain what the client can expect.

An 85-year-old client is alert and able to participate in care. The nurse understands that, according to Erikson, a person's adjustment to the period of senescence will depend largely on adjustment to which developmental stage? 1 Industry versus inferiority 2 Identity versus role confusion 3 Generativity versus stagnation 4 Autonomy versus shame/doubt

3 Generativity versus stagnation

A client is placed on a stretcher and restrained with straps while being transported to the x-ray department. A strap breaks, and the client falls to the floor, sustaining a fractured arm. Later the client shows the strap to the nurse manager, stating, "See, the strap is worn just at the spot where it snapped." What is the nurse's accountability regarding this incident? 1 Exempt from any lawsuit because of the doctrine of respondeat superior 2 Totally responsible for the obvious negligence because of failure to report defective equipment 3 Liable, along with the employer, for misapplication of equipment or use of defective equipment that harms the client 4 Exonerated, because only the hospital, as principal employer, is responsible for the quality and maintenance of equipment

3 Liable, along with the employer, for misapplication of equipment or use of defective equipment that harms the client

A senior high school student, whose immunization status is current, asks the school nurse which immunizations will be included in the precollege physical. Which vaccine should the nurse tell the student to expect to receive? 1 Hepatitis C (HepC) 2 Influenza type B (HIB) 3 Measles, mumps, rubella (MMR) 4 Diphtheria, tetanus, pertussis (DTaP)

3 Measles, mumps, rubella (MMR) Individuals born after 1957 should receive one additional dose of MMR vaccine if they are students in postsecondary educational institutions. Currently there is no vaccine for hepatitis C. The HIB immunization is unnecessary. If the student received an additional DTaP at age 12, it is not necessary. A booster dose of tetanus toxoid (Td) should be received every 10 years.

A nurse is helping a client who observes the traditional Jewish dietary laws to prepare a dietary menu. What considerations should the nurse make? 1 Eating beef and veal is prohibited. 2 Consumption of fish with scales is forbidden. 3 Meat and milk at the same meal are forbidden. 4 Consuming alcohol, coffee, and tea are prohibite

3 Meat and milk at the same meal are forbidden. Jewish dietary laws prohibit any combination of milk and meat at the same meal. The Hindu, not Jewish, religion prohibits the ingestion of beef and veal; many Hindus believe that the cow is sacred. Fish that have scales and fins are considered clean, and therefore allowed in the diet. Seventh Day Adventists, Baptists, Mormons, and Muslims prohibit some or all of these beverages.

A nurse is caring for a client who has a Hemovac portable wound suction device after abdominal surgery. What is the reason why the nurse empties the device when it is half full? 1 Emptying the unit is safer when it is half full. 2 Accurate measurement of drainage is facilitated. 3 Negative pressure in the unit lessens as fluid accumulates, interfering with further drainage. 4 Fluid collecting in the unit exerts positive pressure, forcing drainage back up the tubing and into the wound

3 Negative pressure in the unit lessens as fluid accumulates, interfering with further drainage. As drainage collects and occupies space, the original level of negative pressure decreases; the less the negative pressure, the less effective the drainage. A portable wound suction device is easy and safe to empty regardless of the amount of drainage in the unit. Drainage can be measured accurately by the calibrations on the unit or in a calibrated container after emptying. A one-way valve between the tubing and the collection chamber prevents drainage from entering the tubing and causing trauma to the wound.

The nurse caring for a client with a systemic infection is aware that the assessment finding that is most indicative of a systemic infection is: 1 White blood cell (WBC) count of 8200/mm3 2 Bilateral 3+ pitting pedal edema 3 Oral temperature of 101.3º F 4 Pale skin and nail beds

3 Oral temperature of 101.3º F An elevated temperature of 101.3° F is most indicative of a systemic infection. A white blood cell (WBC) count of 8200/mm3 is within the WBC normal range of 5000-10,000/mm3. Pedal edema is generally not related to an infectious process. Pale skin and nail beds may be related to an infectious process but not necessarily.

When nurses are conducting health assessment interviews with older clients, they should: 1 Leave a written questionnaire for clients to complete at their leisure 2 Ask family members rather than the client to supply the necessary information 3 Spend time in several short sessions to elicit more complete information from the clients 4 Keep referring to previous questions to ascertain that the information given by clients is correct

3 Spend time in several short sessions to elicit more complete information from the clients Spending time in several short sessions reduces client fatigue and compensates for a shortened attention span, which is common in the older adult. The questionnaire may never be completed if it is left for the client to complete at his or her leisure. Asking family members rather than the client to supply the necessary information is degrading to the client; the client should be asked initially and, if necessary, family can be asked to fill in details later. Constantly referring to previous questions may be overwhelming and create feelings of anger and resentment.

To prevent footdrop in a client with a leg cast, the nurse should: 1 Encourage complete bed rest to promote healing of the foot. 2 Place the foot in traction. 3 Support the foot with 90 degrees of flexion. 4 Place an elastic stocking on the foot to provide support.

3 Support the foot with 90 degrees of flexion. To prevent footdrop (plantar flexion of the foot due to weakness or paralysis of the anterior muscles of the lower leg) in a client with a cast, the foot should be supported with 90 degrees of flexion. Bed rest can cause footdrop, and 45 degrees is not enough flexion to prevent footdrop . Applying an elastic stocking for support also will not prevent footdrop; a firmer support is required.

A client comes to the clinic complaining of a productive cough with copious yellow sputum, fever, and chills for the past two days. The first thing the nurse should do when caring for this client is to: 1 Encourage fluids 2 Administer oxygen 3 Take the temperature 4 Collect a sputum specimen

3 Take the temperature Baseline vital signs are extremely important; physical assessment precedes diagnostic measures and intervention. This is done after the health care provider makes a medical diagnosis; this is not an independent function of the nurse. Encouraging fluids might be done after it is determined whether a specimen for blood gases is needed; this is not usually an independent function of the nurse. Oxygen is administered independently by the nurse only in an emergency situation. A sputum specimen should be obtained after vital signs and before administration of antibiotics.

A nurse is teaching staff members about the legal terminology used in child abuse. What definition of battery should the nurse include in the teaching? 1 Maligning a person's character while threatening to do bodily harm. 2 A legal wrong committed by one person against property of another. 3 The application of force to another person without lawful justification. 4 Behaving in a way that a reasonable person with the same education would not.

3 The application of force to another person without lawful justification. Battery means touching in an offensive manner or actually injuring another person. Battery refers to actual bodily harm rather than threats of physical or psychological harm. Battery refers to harm against persons instead of property. Behaving in a way that a reasonable person with the same education would not is the definition of negligence.

A client has been diagnosed as "brain dead". The nurse understands that this means that the client has: 1 no spontaneous reflexes. 2 shallow and slow breathing. 3 no cortical functioning with some reflex breathing. 4 deep tendon reflexes only and no independent breathing.

3 no cortical functioning with some reflex breathing. A client who is declared as being brain dead has no function of the cerebral cortex and a flat EEG. The client may have some spontaneous breathing and a heartbeat. The guidelines established by the American Association of Neurology include coma or unresponsiveness, absence of brainstem reflexes, and apnea. There are specific assessments to validate the findings. The other answer options do not fit the definition of "brain dead."

A client with a head injury underwent a physical examination. The nurse observes that the client's temperature assessments do not correspond with the client's condition. An injury to which part of the brain may be the reason for this condition? 1 Pons 2 Medulla 3 Thalamus 4 Hypothalamus

4

A community healthcare nurse is conducting a survey about homeless children in the community. Which finding helps the nurse distinguish absolute homelessness from relative homelessness? 1 The children are under-immunized and at a risk for childhood illnesses. 2 The children are more likely to drop out of school and become unemployable. 3 The children have access to healthcare only through the emergency department. 4 The children do not have a physical shelter and may sleep outdoors or in vehicles.

4

A nurse applies a cold pack to treat an acute musculoskeletal injury. Cold therapy decreases pain by doing what? 1 Promoting analgesia and circulation 2 Numbing the nerves and dilating the blood vessels 3 Promoting circulation and reducing muscle spasms 4 Causing local vasoconstriction, preventing edema and muscle spasms

4

The nurse creates a plan of care for a client with a risk of infection. Which is the most desirable expected outcome for the client? 1 All nursing functions will be completed by discharge. 2 All invasive intravenous lines will remain patent. 3 The client will remain awake, alert, and oriented at all times. 4 The client will be free of signs and symptoms of infection by discharge.

4

The nurse is assessing a client who is undergoing chemotherapy. The nurse notes that the client is using a scarf to cover the head. The nurse asks the client about coping with the altered body image. Which functional pattern does the assessment include? 1 Value-belief pattern 2 Role-relationship pattern 3 Cognitive-perceptual pattern 4 Self-perception-Self-tolerance pattern

4

The nurse is assessing a client with arthritis. Which statement made by the client indicates a precipitating factor that is an intellectual standard for critical thinking? 1 "The pain is usually present in my fingers and knees." 2 "I observed swelling and redness near the pain area." 3 "I feel the pain in each and every joint of my hands and legs." 4 "I run for 30 minutes every day; this exercise increases my pain."

4

The nurse is aware that the nursing diagnosis should follow the North American Nursing Diagnosis Association International (NANDA-I) label. How should the nurse document the nursing diagnosis in a three-part format? 1 NANDA-I label, related factor, and etiologies 2 NANDA-I label, risk factor, and nursing interventions 3 NANDA-I label, related factor, and nursing interventions 4 NANDA-I label, related factor, and defining characteristics

4

Which activity would the nurse explain can be performed by infants of aged 6 to 8 months? 1 Holding a pencil 2 Showing hand preference 3 Placing objects into containers 4 Transferring objects from hand to hand

4

Which activity would the nurse use as an example of fine motor skills of infants aged 2 to 4 months? 1 Turning from side to back 2 Sitting erect using support 3 Showing good head control 4 Bringing objects from hand to mouth

4

Which integumentary finding is related to skin texture? 1 Elasticity 2 Vascularity 3 Fluid buildup 4 Character of the surface

4

Which nursing process involves delegation and verbal discussion with the healthcare team? 1 Planning 2 Evaluation 3 Assessment 4 Implementation

4

While performing a physical assessment, the nurse notices a minute, nonpalpable change in the skin color of a client. What might be the type of skin lesion involved? 1 Wheal 2 Papule 3 Vesicle 4 Macule

4

A nurse is caring for a client that has been admitted with right sided heart failure. The nurse notes that the client has dependent edema around the area of the feet and ankles. In order to characterize the severity of the edema, the nurse presses the medial malleolus area and notes an 8 mm depression after release. This nurse understands that the edema should be documented as: 1 1+ 2 2+ 3 3+ 4 4+

4 4+ Dependent edema around the area of feet and ankles often indicates right sided heart failure or venous insufficiency. The nurse should assess for pitting edema by pressing firmly for several seconds then release to assess for any depression left on the skin. The grading of 1+ to 4+ characterizes the severity of the edema. A grade of grade of 4+ indicates an 8 mm depression. A grade of 1+ indicates a 2 mm depression. A grade of 2 + indicates a 4 mm depression. A grade of 3+ indicates a 6 mm depression.

The nurse instructs a client with a new colostomy to avoid foods and drinks that produce a large amount of gas, and specifically to avoid the intake of: 1 Milk 2 Cheese 3 Coffee 4 Cabbage

4 Cabbage Cabbage is a gas-producing food that can cause a client with a colostomy problems with odor control and ballooning of the ostomy bag, which may break the device seal and allow leakage. Milk, cheese, and coffee, in moderation, should not cause excessive gas problems. The client with a new colostomy should slowly introduce new foods into the diet to test toleration.

While assessing an immobilized client, the nurse notes that the client has shortened muscles over a joint, preventing full extension. This condition is known as: 1 Osteoarthritis 2 Osteoporosis 3 Muscle atrophy 4 Contracture

4 Contracture

A nurse is caring for a postoperative client who had general anesthesia during surgery. What independent nursing intervention may prevent an accumulation of secretions? 1 Postural drainage 2 Cupping the chest 3 Nasotracheal suctioning 4 Frequent changes of position

4 Frequent changes of position

Refusing to follow the prescribed treatment regimen, a client plans to leave the hospital against medical advice. The nurse recognizes that it is important to inform the client that he or she: 1 Is acting irresponsibly. 2 Is violating the hospital policy. 3 Must obtain a new healthcare provider for future medical needs. 4 Must accept full responsibility for possible undesirable outcomes

4 Must accept full responsibility for possible undesirable outcomes

A client is being admitted to a medical unit with a diagnosis of pulmonary tuberculosis. The nurse should assign the client to which type of room? 1 Private room 2 Semi-private room 3 Room with windows that can be opened 4 Negative airflow room

4 Negative airflow room Tuberculosis is an airborne contagious disease that is best contained in a negative airflow room. Negative airflow rooms are always private. A private room, semiprivate room, and a room with windows that can be opened are not appropriate for the standard of care for a client diagnosed with tuberculosis. Additionally, opening windows would present a possible safety hazard in a client's room.

The nurse is caring for a surgical client who develops a wound infection during hospitalization. How is this type of infection classified? 1 Primary 2 Secondary 3 Superinfection 4 Nosocomial

4 Nosocomial A nosocomial infection is acquired in a health care setting. This is also referred to as a hospital-acquired infection. It is a result of poor infection control procedures such as failure to wash hands between clients. A primary infection is synonymous with initial infection. A secondary infection is made possible by a primary infection that lowers the host's resistance and causes an infection by another kind of organism. A superinfection is a new infection caused by an organism different from that which caused the initial infection. The microbe responsible is usually resistant to the treatment given for the initial infection.

A nurse is caring for an elderly client who has constipation. Which independent nursing intervention helps to reestablish normal bowel pattern? 1 Administer a mineral oil enema. 2 Offer one cup of fluid every hour. 3 Manually remove fecal impactions. 4 Offer a cup of prune juice.

4 Offer a cup of prune juice. Prune juice does not require a health practitioner order and helps to promote bowel movement because it contains sorbitol, which increases water retention in feces

A nurse must establish and maintain an airway in a client who has experienced a near-drowning in the ocean. For which potential danger should the nurse assess the client? 1 Alkalosis 2 Renal failure 3 Hypervolemia 4 Pulmonary edema

4 Pulmonary edema Additional fluid from surrounding tissues will be drawn into the lung because of the high osmotic pressure exerted by the salt content of the aspirated ocean water; this results in pulmonary edema. Hypoxia and acidosis may occur after a near-drowning. Renal failure is not a sequela of near-drowning. Hypovolemia occurs because fluid is drawn into the lungs by the hypertonic saltwater.

A client using fentanyl (Duragesic) transdermal patches for pain management in late-stage cancer dies. What should the hospice nurse who is caring for this client do about the patch? 1 Tell the family to remove and dispose of the patch. 2 Leave the patch in place for the mortician to remove. 3 Have the family return the patch to the pharmacy for disposal. 4 Remove and dispose of the patch in an appropriate receptacle

4 Remove and dispose of the patch in an appropriate receptacle

The nurse prepares to give a prescribed capsule of hydroxyzine (Vistaril) to a client. The client begins to vomit so the nurse holds the oral medication. The nurse has not opened the medication package. Proper and safe disposal of the capsule of hydroxyzine requires the nurse to: 1 Drop the capsule into the sharps container 2 Crush the capsule and flush it into the sewer system 3 Place the capsule into a red biohazard bag and tie it shut 4 Return the capsule to the pharmacy

4 Return the capsule to the pharmacy Medication taken from a stock supply cannot be returned; it should be returned to the pharmacy for safe disposal.

A client is hospitalized because of severe depression. The client refuses to eat, stays in bed most of the time, does not talk with family members, and will not leave the room. The nurse attempts to initiate a conversation by asking questions but receives no answers. The nurse is frustrated and tells the client that if there is no response, the nurse will leave and the client will remain alone. How should the nurse's behavior be interpreted? 1 A system of rewards and punishment is being used to motivate the client. 2 Leaving the client alone allows time for the nurse to think of other strategies. 3 This behavior indicates the client's desire for solitude that the nurse is respecting. 4 This threat is considered assault, and the nurse should not have reacted in this manner.

4 This threat is considered assault, and the nurse should not have reacted in this manner. This response is a threat (assault) because the nurse is attempting to put pressure on the client to speak or be left alone. This is not a reward and punishment technique that is used in behavior modification therapy. Clients in emotional crisis should not be left alone.

Which theories are most relevant to development in adults? Select all that apply. 1 Piaget's theory 2 Erikson's theory 3 Kohlberg's theory 4 Stage-Crisis theory 5 Life Span approach

4, 5

The nurse is monitoring a client's hemoglobin level. The nurse recalls that the amount of hemoglobin in the blood has what effect on oxygenation status?

A low hemoglobin level causes reduced oxygen-carrying capacity.

Arrange the sequence of events occurring during a fever secondary to pyrogens in chronological order.

A true fever results from an alteration in the hypothalamic set point. Pyrogens act as antigens that trigger the immune system response. The hypothalamus reacts by raising the set point, thereby increasing the body temperature. Once the pyrogens are removed, the third phase of a febrile episode occurs. Heat loss responses are initiated when the hypothalamus set point drops.

What are the important points to be considered when imparting practical knowledge to nursing students about preventing complications in the hospital? Select all that apply. A. Nursing students are not accountable if a client is harmed. B. Nursing students should never be assigned any tasks they are unprepared for. C. Nursing students are employees of the hospital and may act as witnesses to consent forms. D. Nursing students can work as nursing assistants or nurse's aides when not attending classes. E. Nursing students should notify the nursing supervisor in case they are delegated tasks they are not prepared for.

A, D, and E. Nursing students should never be assigned to perform tasks for which they are unprepared. A nursing student can work as a nursing assistant or a nurse's aide when not attending classes. In case anyone instructs a nursing student to perform a task that he or she is unprepared for, the nursing supervisor should be notified as soon as possible. A nursing student is indeed accountable if a client is harmed. A nursing student is not an employee of the hospital and he or she cannot act as a witness to consent forms because these forms are legal documents.

A nursing student is listing the points that need to be remembered regarding the United Network for Organ Sharing (UNOS) program. Which point listed by the nursing student is accurate? A. "The United Network for Organ Sharing (UNOS) has a contract with the federal government." B. "The United Network for Organ Sharing (UNOS) protects the donor's estate from liability for injury or damage." C. "The United Network for Organ Sharing (UNOS) gives priority to international clients who need organs on an urgent basis." D. "The United Network for Organ Sharing (UNOS) provides civil and criminal immunity to the hospital and the primary healthcare provider."

A. "The United Network for Organ Sharing (UNOS) has a contract with the federal government." The United Network for Organ Sharing (UNOS) has a contract with the federal government. The National Organ Transplant Act of 1984 protects the donor's estate from liability for injury or damage. The United Network for Organ Sharing gives priority to clients in their geographical area who need organs on an urgent basis. The National Organ Transplant Act of 1984 provides civil and criminal immunity to the hospital and the healthcare provider.

A registered nurse is educating a nursing student about the stages of changes in a client's health behavior. Which statement describes the stage of contemplation? A. "The client considers a change within the next 6 months." B. "The client does not intend to make changes within the next 6 months." C. "The client is actively engaged in strategies to change D. "The client displays sustained change over time; this begins 6 months after action has started and continues indefinitely."

A. "The client considers a change within the next 6 months." In the contemplation stage, the client considers a change within the next 6 months. In the precontemplation stage, the client does not intend to make changes within the next 6 months. In the action stage, the client is actively engaged in strategies to change behavior. This stage lasts up to 6 months. When sustained change is noticed over time and begins 6 months after action has started and continues indefinitely, the client has reached the maintenance stage.

The professional obligation of a nurse to assume responsibility for actions is referred to as what? A. Accountability B. Individuality C. Responsibility D. Bioethics

A. Accountability Nurses have an obligation to uphold the highest standards of practice, assume full responsibility for actions, and maintain quality in the knowledge base and skill of the profession; this is referred to as accountability. Individuality and responsibility are positive characteristics of the nurse but are not necessarily professional obligations. Bioethics is a field of study concerned with the ethics and philosophical implications of certain biologic and medical procedures and treatments.

A client tells the nurse, "I keep reverting to my old habit of drinking soda, although I have stopped drinking as much." What stage of health behavior change has the client reached? A. Action stage B. Preparation stage C. Maintenance stage D. Contemplation stage

A. Action stage The client in this situation has reached the action stage of health behavior change. In this stage, old habits may get in the way of new behaviors. In the preparation stage, the client understands that the advantages of the health behavior change outweigh its disadvantages. In this situation, the client has already made changes in health behavior. In the maintenance stage, the client continues the health behavior change indefinitely. In the contemplation stage, the client may be ambivalent but is more ready to accept information regarding health behavior change.

What purpose does block and parish nursing serve in preventive and primary care services? A. Block and parish nursing provides services to older clients or those who are unable to leave their homes. B. Block and parish nursing provides primary care to a specific client population that lives in a specific community. C. Block and parish nursing provides nursing services with a focus on health promotion and education as well as on chronic disease. D. Block and parish nursing provides services aimed at increasing worker productivity, decreasing absenteeism, and reducing the use of expensive medical care.

A. Block and parish nursing provides services to older clients or those who are unable to leave their homes. Block and parish nursing provides services to older clients or those who are unable to leave their homes. Community health centers provide primary care to a specific client population living in a specific community. Nurse-managed clinics provide nursing services with a focus on health promotion and education as well as on chronic disease. Occupational health services provide services that aim to increase worker productivity, decrease absenteeism, and reduce the use of expensive medical care.

What purpose does a community health center serve in preventive and primary care services? A. Community health centers are outpatient clinics that provide primary care to a specific population. B. Community health centers aim to increase worker productivity, decrease absenteeism, and reduce the use of costly medical care. C. Community health centers emphasize program management, interdisciplinary collaboration, and community health principles. D. Community health centers include a complete program designed for health promotion and accident or illness prevention in the workplace.

A. Community health center are outpatient clinics that provide primary care to a specific population. Community health centers are outpatient clinics that provide primary care to a specific population, such as clients with young children or clients with diabetes. Occupational health services aim to increase worker productivity, decrease absenteeism, and reduce the use of costly medical care. School health services emphasize program management, interdisciplinary collaboration, and community health principles. Occupational health services include a complete program designed for health promotion and accident or illness prevention in the workplace.

An octogenarian client asks the nurse about the United States' government-funded national health insurance program. About which healthcare plan should the nurse inform the client? A. Medicare B. Long-term care insurance C. Private insurance D. Preferred provider organization

A. Medicare Medicare is a government-funded national health insurance program for people 65 years and older in the United States. The nurse should inform the client about this plan. Long-term care insurance is a supplemental insurance for the coverage of long-term care services. Private insurance is a traditional fee-for-service plan in which the payment is computed on basis of the number of services used after the client is treated. A preferred provider organization is a type of managed care plan that limits an enrollee's choice to a list of preferred hospitals, physicians, and providers.

A nursing student is recalling the definition of Nurse Practice Acts. What do the Nurse Practice Acts do? A. Nurse Practice Acts describe and define the legal boundaries of nursing practice within each state. B. Nurse Practice Acts reflect the knowledge and skills possessed by nurses practicing in their profession. C. Nurse Practice Acts are legal requirements that describe the minimum acceptable nursing care. D. Nurse Practice Acts protect individuals from losing their health insurance when changing jobs by providing portability.

A. Nurse Practice Acts describe and define the legal boundaries of nursing practice within each state. The Nurse Practice Acts describe and define the legal boundaries of nursing practice within each state. They help to distinguish between nursing and medical practice and establish education and licensure requirements for nurses. Standards of care reflect the knowledge and skills possessed by nurses who are active practitioners in their profession. Standards of care are legal requirements that define the minimum acceptable nursing care. The Health Insurance Portability and Accountability Act (HIPAA) protects individuals from losing their health insurance when changing jobs by providing portability.

A nursing student is listing the different aspects of the healthcare services pyramid. Under which type of healthcare services should the nursing student include family planning? A. Primary care B. Continuing care C. Restorative care D. Secondary Acute care

A. Primary Care The nursing student should include family planning under primary care. Family planning is not a part of continuing care, restorative care or secondary acute care healthcare services.

A registered nurse is educating a nursing student about the process of resolving an ethical dilemma. What information should the nurse provide regarding negotiation of outcomes? A. "A nurse should provide a personal point of view." B. "Negotiations should be held in formal settings only." C. "Negotiation takes place immediately after gathering information." D. "The group agrees to a statement of the problem during the negotiation process."

AA. "A nurse should provide a personal point of view." During the process of negotiating outcomes, the nurse is required to provide a personal point of view. Negotiations may take place informally at the client's bedside or in a formal setting. After gathering relevant information regarding an ethical dilemma, the nurse is required to examine his or her own values and formulate an opinion regarding the matter. When verbalizing the problem, the group agrees to a statement of the problem to begin discussions. This step is performed before negotiating outcomes. Negotiations take place after determining all possible courses of action.

Which is used for determining the hours of care and staff required for a group of clients?

Acuity records

A hospitalized client is scheduled to have a sigmoidoscopy. The nurse anticipates that pre-procedure prescriptions will include: 1 Providing instructions about restraints used during the procedure. 2 Administering a fleet enema 1 hour before the procedure. 3 Encouraging increased intake of clear fluids. 4 Administering morphine 30 minutes before the procedure.

Administering a fleet enema 1 hour before the procedure. To facilitate visualization of the rectum and the sigmoid colon, the lower colon must be emptied immediately before the procedure. A fleet or tap water enema should be used. The client will be kept NPO for at least 8 hours before the procedure. Morphine is not typically used as a pre-op medication before a sigmoidoscopy. Restraints are not typically used during the procedure.

The nurse is caring for a client who is in pain following surgery. The nurse informs the primary health care provider about the client's request for pain medication. What is the role of the nurse in this situation?

Advocate

When caring for a client with varicella and disseminated herpes zoster, the nurse should implement which types of precautions?

Airborne Contact Standard

A nurse is teaching a parent about the different temperaments that a child may display. What temperament would an easy child display? Select all that apply.

An easy child is open and adaptable to change An easy child is regular and predictable in his or her habits. An easy child displays a mild-to-moderately intense mood that is typically positive.

The nurse assesses a client's eyes and notices that the pupils are unequal in size and constricted. What condition would be found in the client's medical record?

Anisocoria

The nurse is caring for a client who survived a severe burn injury. Which action should the nurse perform immediately based on priority?

Assessing airway patency

Which physical assessment maneuver is the nurse performing when instructing the client to breathe in slowly and a little more deeply than normal through the mouth?

Auscultation

A nurse questions the staff about a change in a client's plan of care. What does this demonstrate?

Authority

In what ways can a nurse prevent medication errors?

Avoid using abbreviations and acronyms Minimize the use of verbal and telephone orders Check three times before giving a drug by comparing the drug order and medication profile

What services do community health centers provide in preventive and primary care services? Select all that apply. A. Day care B. Health screenings C. Physical assessments D. Disease management E. Acute and chronic care management

B, C, and D Health screenings, physical assessments, and disease management services are provided by community health centers in preventive and primary care services. Day care and acute and chronic care management services are provided by nurse-managed clinics.

A primary healthcare provider notes that all conventional treatment procedures have proved to be ineffective in managing a client's disorder. The primary healthcare provider decides to try an experimental treatment. The nurse ensures that the client has understood the implications of the new treatment plan thoroughly and then signs the client's consent form as a witness. Which basic healthcare ethic does the nurse follow in this situation? A. Justice B. Autonomy C. Beneficence D. Nonmaleficence

B. Autonomy Autonomy refers to the commitment to include clients in decisions about all aspects of care as a way of acknowledging and protecting their independence. In the given situation, the nurse ensures that the client has thoroughly understood the new treatment plan before gaining written consent. This ensures that the client is involved in the decision-making process appropriately. Justice refers to fairness. The given situation does not deal with fairness. Beneficence refers to taking positive actions to help others. This involves keeping the interests of the client before self-interest. Nonmaleficence is the avoidance of harm or hurt. Weighing the pros and cons of the new treatment plan would involve nonmaleficence.

An unemancipated pediatric client is to undergo a medical procedure. Who is the appropriate authority to provide consent? A. The court B. Either of the child's parents C. One of the child's grandparents D. The parent who holds legal custody

B. Either of the child's parents For unemancipated pediatric clients, the parents are required to provide consent. Either parent may do so. The court intervenes in situations where the parents refuse to allow a child to undergo treatment. A grandparent may provide consent only if the situation is an emergency and the parents are not present. If the parents are divorced, a parent with legal custody of the child may provide consent.

What necessary information should the nurse remember regarding assisted suicide, according to American Nurses Association (ANA) (2008)? A. Nurses may have an open attitude toward the client's end of life. B. Nurses' participation in assisted suicide may violate the code of ethics. C. Nurses may listen to the client's expressions of fear and to attempt to control the client's pain. D. Nurses can participate in assisted suicide only if the individual could make an oral and written request.

B. Nurses' participation in assisted suicide may violate the code of ethics. According to the ANA, a nurse's participation in assisted suicide will violate their code of ethics. According to the American Association of Colleges of Nursing (AACN) and the International Council of Nurses', the nurse may have an open attitude toward the client's end of life. According to the American Association of Colleges of Nursing (AACN) and the International Council of Nurses', nurses may listen to the client's expressions of fear and to attempt to control the client's pain. According to the Oregon Death with Dignity Act (1994) the primary health care provider in the state of Oregon can participate in assisted suicide only if an individual with terminal disease makes an oral and written request to end his or her life in a humane and dignified manner.

What does the Emergency Medical Treatment and Active Labor Act (EMTALA) of 1986 state? A. The act states that employees can change jobs without losing health insurance coverage. B. The act states that the hospital should not discharge or transfer a client if an emergency condition exists. C. The act states that the hospital protects a person who is HIV positive but does not have acquired immunodeficiency syndrome (AIDS). D. The act states that health care institutions should provide written information to clients concerning their rights under state law to make decisions.

B. The act states that the hospital should not discharge or transfer a client if an emergency condition exists. According to the Emergency Medical Treatment and Active Labor Act (EMTALA) act, the hospital should not discharge or transfer a client if an emergency condition exists. According to the Health Insurance Portability and Accountability Act of 1996 (HIPAA), employees can change jobs without losing health insurance coverage. According to the Americans with Disabilities Act (ADA) of 1990, the hospital protects a person who is HIV positive but does not have acquired immunodeficiency syndrome (AIDS). According to the Patient Self-Determination Act (PSDA) of 1991, healthcare institutions should provide written information to clients about their rights under state law to make decisions.

A nurse is teaching members of a health care team how to help disabled clients stand and transfer from the bed to a chair. To protect the caregivers from injury, the nurse teaches them to lift the client by first placing their arms under the client's axillae and doing what next

Bending and then straightening their knees

The nurse is caring for a client that underwent a rhinoplasty surgical procedure 5 hours ago. After administering pain medication, the nurse notes the client is swallowing frequently. The nurse understands that the cause of frequent swallowing is most likely caused from: 1 A normal response to the analgesic 2 Oral dryness caused by nasal packing 3 An adverse reaction to anesthesia 4 Bleeding posterior to the nasal packing

Bleeding posterior to the nasal packing Frequent swallowing may indicate bleeding in the posterior pharynx. Oral dryness causes thirst, not an increase in swallowing. Frequent swallowing is not a normal response to rhinoplasty or analgesics/anesthesia.

A registered nurse is caring for a client who is on isolation precautions. Which tasks can be safely assigned to the nursing assistive personnel?

Bringing equipment to the client's room Transporting the client to a diagnostic test

How does a nurse play the role of a "change agent" in a community-based nursing practice?

By identifying and implementing new and more effective approaches to problems

A nurse is helping a client to maintain and regain health, manage his or her disease and symptoms, and attain a maximal level of function and independence through the healing process. What role is the nurse playing? A. Manager B. Advocate C. Caregiver D. Communicator

C. Caregiver As a caregiver, a nurse helps clients maintain and regain health, manage diseases and symptoms, and attain a maximal level of function and independence through the healing process. As a manager, the nurse coordinates the activities of members of the nursing staff in delivering nursing care and has personnel, policy, and budgetary responsibility for a specific nursing unit or agency. As a client's advocate, the nurse protects the client's human and legal rights and provides assistance in asserting these rights if the need arises. As a communicator, the nurse learns about a client's strengths and weaknesses and his or her needs through effective communication.

A nurse is hired to work in a healthcare facility that has a completely computer-based client information system. The nurse in charge knows that the newly hired nurse is knowledgeable about this system when the nurse says what? A. "More medication errors are made when this system is used." B. "It is disappointing that nurses are not allowed to use this system." C. "Client information is immediately available when this system is used." D. "I will have less time to provide direct care to my clients with this system."

C. Client information is immediately available when this system is used. The intent of these systems is to streamline documentation and record keeping for all appropriate health team members, including nurses. There is a reduction in medication errors with this type of system. Data are immediately available to appropriate health team members without the need to depend on record or chart availability. By streamlining documentation and record keeping, these systems increase opportunities for more direct client care by nurses.

A client is placed on a stretcher and restrained with straps while being transported to the x-ray department. A strap breaks, and the client falls to the floor, sustaining a fractured arm. Later the client shows the strap to the nurse manager, stating, "See, the strap is worn just at the spot where it snapped." What is the nurse's accountability regarding this incident? A. Exempt from any lawsuit because of the doctrine of respondeat superior B. Totally responsible for the obvious negligence because of failure to report defective equipment C. Liable, along with the employer, for misapplication of equipment or use of defective equipment that harms the client D. Exonerated, because only the hospital, as principal employer, is responsible for the quality and maintenance of equipment

C. Liable, along with the employer, for misapplication of equipment or use of defective equipment that harms the client. Using a stretcher with worn straps is negligent; this oversight does not reflect the actions of a reasonably prudent nurse. The nurse is responsible and must ascertain the adequate functioning of equipment. The hospital shares responsibility for safe, functioning equipment.

A registered nurse is teaching an economically challenged client about different healthcare plans. Which healthcare plan is the most expensive? A. Medicaid B. Medicare C. Long-term care insurance (LTC) D. Managed care organization (MCO)

C. Long-term care insurance (LTC) The long-term care insurance (LTC) plan is the most expensive insurance plan.

A client with cancer is undergoing treatment in a hospital. The nurse finds the orders from the primary healthcare provider inappropriate. Clarification from the healthcare provider does not resolve the nurse's doubts. Who should the nurse contact and inform next? A. Risk manager B. Nursing student C. Supervising nurse D. Nurse administrator

C. Supervising nurse The nurse should go to the supervising nurse or follow the established chain of command if he or she finds any discrepancies in the primary healthcare provider's orders. All nurses must act as risk managers, depending upon the situation. The nurse in question should follow the established chain of command to address his or her doubts. A nursing student is still a novice and is too inexperienced to handle such matters. A nurse administrator manages client care and the delivery of specific nursing services within a healthcare agency; a nurse administrator is not the appropriate person to ask for help in solving the problem at hand.

What should a nurse understand regarding the Uniform Anatomical Gift Act? A. The Uniform Anatomical Gift Act prohibits the purchase or sale of organs. B. The Uniform Anatomical Gift Act protects the rights of people with physical or mental disabilities. C. The Uniform Anatomical Gift Act gives individuals who have reached 18 years of age the right to make an organ donation. D. The Uniform Anatomical Gift Act provides civil and criminal immunity to hospitals and healthcare providers who adhere to the act.

C. The Uniform Anatomical Gift Act gives individuals who have reached 18 years of age the right to make an organ donation. According to the Uniform Anatomical Gift Act, individuals who are at least 18 years of age have the right to make organ donations. The National Organ Transplant Act of 1984 prohibits the purchase or sale of organs. The Americans with Disabilities Act (1990) protects the rights of people with physical or mental disabilities. The National Organ Transplant Act of 1984 provides civil and criminal immunity to hospitals and healthcare providers.

When should a medical examiner decide whether a postmortem examination should be conducted? A. When a client dies under normal circumstances B. When a client dies after 48 hours of admission to the hospital C. When a client dies within 24 hours of admission to the hospital D. When the client gives a written consent to perform autopsy before death

C. When a client dies within 24 hours of admission to the hospital. If a client dies within 24 hours of admission to the hospital, the medical examiner is required to decide whether a postmortem examination should be conducted. If a client dies under suspicious circumstances, the medical examiner decides whether a postmortem examination is necessary. The medical examiner does not make the decision regarding postmortem examination if the client dies after 48 hours of being admitted to the hospital. A client may give a written consent before death to perform an autopsy. Such instances are not subject to the review of the medical examiner.

A hospital needs to hire a nursing staff for the intensive care of cancer clients. Which of these positions is most likely to be filled by the nurse?

Clinical nurse specialist

Which interview technique is the nurse using when asking a client to score the pain on a scale from 0 to 10?

Closed-ended questioning

What finding would be consistent with long-standing hypoxemia in a client who reports shortness of breath?

Clubbing

With reference to the nursing process as a system, what is content?

Content is the product and information obtained from the system.

A nurse is reinforcing teaching to an adolescent about type 1 diabetes and self-care. Which questions from the client indicate a need for additional teaching in the cognitive domain? (Select all that apply.) 1 "What is diabetes?" 2 "What will my friends think?" 3 "How do I give myself an injection?" 4 "Can you tell me how the glucose monitor works?" 5 "How do I get the insulin from the vial into the syringe?"

Correct 1 "What is diabetes?" Correct 4 "Can you tell me how the glucose monitor works?" Acquiring knowledge or understanding aids in developing concepts, rather than skills or attitudes, and is a basic learning task in the cognitive domain. Values and self-realization are in the affective domain. Skills acquisition is in the psychomotor domain.

A pharmacy technician arrives on the nursing unit to deliver opioids and, following hospital protocol, asks the nurse to receive the medications. The nurse is assisting a confused and unsteady client back to the client's room. How should the nurse respond to the technician? 1 "I can't receive them right now. Please wait a few minutes or come back." 2 "Please leave the medications and sign-out sheet in a location where I can see them." 3 "Please bring them to me and I will be sure to put them away in a couple of minutes." 4 "I can't receive them right now. Please give them to the unlicensed assistive personnel (UAP)."

Correct1 "I can't receive them right now. Please wait a few minutes or come back." The transfer of controlled substances from one authorized person to another must occur according to protocol. In this situation the controlled substance must be returned to the pharmacy and delivered at a later time. The controlled substances cannot be left unattended. The nurse cannot delay the securing of controlled substances; if time is not available when the medications are delivered, they must be returned to the pharmacy. The UAP does not have the authority to receive controlled substances.

How can a nurse best evaluate the effectiveness of communication with a client? 1 Client feedback 2 Medical assessments 3 Health care team conferences 4 Client's physiologic responses

Correct1 Client feedback

The nurse is preparing to reinforce teaching a client about self-injection of insulin. Which action by the nurse will increase the effectiveness of the teaching session? 1 Wait until a family member is also present. 2 Assess the client's barriers to learning self-injection techniques. t3 Begin with simple written instructions describing the technique. 4 Wait until the client has accepted the new diagnosis of Type 1 Diabetes Mellitus.

Correct2 Assess the client's barriers to learning self-injection

An 85-year-old client has just been admitted to a nursing home. When designing a plan of care for this older adult the nurse recalls the expected sensory losses associated with aging. (Select all that apply.) 1 Difficulty in swallowing 2 Diminished sensation of pain 3 Heightened response to stimuli 4 Impaired hearing of high-frequency sounds 5 Increased ability to tolerate environmental heat

Correct2 Diminished sensation of pain Correct4 Impaired hearing of high-frequency sounds

A client with hemiplegia is staring blankly at the wall and reports feeling like half a person. What is the most appropriate initial nursing action? 1 Use techniques to distract the client. 2 Include the client in decision making. 3 Offer to spend more time with the client. 4 Help the client to problem-solve personal issues

Correct3 Offer to spend more time with the client. Because of the profound effect of paralysis on body image, the nurse should foster an environment that permits exploration of feelings without judgment, punishment, or rejection. Attempts to distract the client may be interpreted as denial of the client's feelings and will not resolve the underlying problem. Including the client in decision making and helping the client to problem-solve personal issues are an important part of nursing care, but they are not related to the client's feelings.

What should a nurse recommend to help a client best during the period immediately after a spouse's death? 1 Crisis counseling 2 Family counseling 3 Marital counseling 4 Bereavement counseling

Correct4 Bereavement counseling Bereavement counseling involves being a part of a group of people who also have sustained a loss; members provide support to each other. Individual counseling will not provide the support that a group provides; group counseling may last longer than crisis intervention. The information provided did not indicate other family members. Marital counseling involves both a husband and a wife.

A client is admitted to the hospital because of multiple chronic health problems. What is the priority nursing intervention at this time? 1 Advising the client to join a support group immediately after discharge. 2 Assuring the family that staff members will take care of the client's needs. 3 Reminding the client to keep medical follow-up appointments after discharge. 4 Conducting a multidisciplinary staff conference early during the client's hospitalization.

Correct4 Conducting a multidisciplinary staff conference early during the client's hospitalization.

When being interviewed for a position as a licensed practical nurse, the applicant is asked to identify an example of an intentional tort. What is the appropriate response? 1 Negligence 2 Malpractice 3 Breach of duty 4 False imprisonment

Correct4 False imprisonment False imprisonment is a wrong committed by one person against another in a willful, intentional way without just cause or excuse. Negligence is an unintentional tort. Malpractice, which is professional negligence, is classified as an unintentional tort. Breach of duty is an unintentional tort.

A nurse is supportive of a child receiving long-term rehabilitation in the home rather than in a health care facility. Why is living with the family so important to a child's emotional development? 1 It provides rewards and punishment. 2 The child's development is supported. 3 It reflects the mores of a larger society. 4 It is where child's identity and roles are learned

Correct4 It is where child's identity and roles are learned Socialization, values, and role definition are learned within the family and help develop a sense of self. Once established in the family, the child can move more easily into society.

A client with a terminal illness reaches the stage of acceptance. How can the nurse best help the client during this stage? 1 Acknowledge the client's crying. 2 Encourage unrestricted family visits. 3 Explain details of the care being given. 4 Stay nearby without initiating conversation

Correct4 Stay nearby without initiating conversation

A nurse helps a client to clarify health problems and choose appropriate courses of action. What competency in community-based practice is the nurse exercising?

Counselor

A nurse is evaluating the appropriateness of a family member's initial response to grief. What is the most important factor for the nurse to consider? 1 Personality traits 2 Educational level 3 Cultural background 4 Past experiences with death

Cultural background In the initial stage of grief the degree of anguish experienced is influenced by cultural background. Although personality traits factor into the grief process, they are not as important as culture. Educational level is not related directly to a grief response. While past experience is important, it is not as significant as culture.

A nursing student is listing the professional responsibilities and roles of the nurse. Who is the most independently functioning nurse? A. Nurse educator B. Nurse researcher C. Nurse administrator D. Advanced practice registered nurse

D. Advanced practice registered nurse. The advanced practice registered nurse is the most independently functioning nurse. The nurse educator, nurse researcher, and nurse administrator all must be associated with an organization to pursue their professional prospects.

A nursing student is listing examples of active and passive health promotion strategies. Which strategy is an example of a passive health promotion strategy? A. Weight-reduction program B. Smoking-cessation program C. Drug abuse prevention strategy D. Fluoridation of municipal drinking water

D. Fluoridation of municipal drinking water. Passive strategies of health promotion help people benefit from the activities of others without direct involvement. The fluoridation of municipal drinking water is an example of a passive health promotion strategy. Active strategies of health promotion require clients to adopt specific programs for improving health. Weight-reduction programs, smoking-cessation programs, and drug abuse prevention strategies are examples of active health promotion activities.

According to the Institute of Medicine (IOM) competencies for the 21st century, what task should the nurse perform when using evidence-based practice? A. Share decision making and management. B. Cooperate, collaborate, and communicate. C. Practice using basic safety design principles. D. Participate in research activities when possible.

D. Participate in research activities when possible. According to IOM competencies for the 21st century, when using evidence-based practice, the nurse should participate in research activities when possible. When providing client-centered care, the nurse should share decision making and management. When working in interdisciplinary teams, the nurse should cooperate, collaborate, and communicate. When applying quality improvement, the nurse should practice using basic safety design principles.

Which intrinsic factors may contribute to falls in older adults?

Deconditioning Impaired vision

Considering Erikson's developmental theories, a 21-year-old male client who has sustained a spinal injury below the level of T6 will most likely have difficulty with: 1 Mastering his environment 2 Identifying with the male role 3 Developing meaningful relationships 4 Differentiating himself from the environment

Developing meaningful relationships Developing meaningful relationships is the young-adult task associated with intimacy versus isolation. Mastering his environment is a toddler's task associated with autonomy versus shame and doubt. Identifying with the male role is a preschool-age child's task associated with initiative versus guilt. Differentiating himself from the environment is a toddler's task associated with autonomy versus shame and doubt.

A client in a hospice home care program is experiencing severe pain. Morphine has been prescribed for pain management. Which information should the nurse plan to explain to the client in preparation for this pain management regimen?

Dosages of the drug will be given automatically at regular intervals around the clock.

In a clinical study, subjects were given chlorhexidine and betadine as antiseptics. How will a nurse researcher categorize this research?

Experimental Research

Which of these programs is least likely to focus on medication delivery process modification?

Experimental research

While teaching a nursing student, a registered nurse says "This is a study in which the investigator controls the study variable and randomly assigns subjects to different conditions to test the variable." Which type of research is the above statement?

Experimental research

A nurse receives a shift report on four adult clients that are between the ages of 25-55. Which client should the nurse assess first? 1 Male client with a hemoglobin of 15.9 2 Female client on warfarin (Coumadin) with an International Normalized Ratio (INR) of 7.5 3 Female client taking daily calcium supplements with a serum calcium level of 9.4 4 Male client with a blood urea nitrogen (BUN) of 20 and a creatinine of 1.1

Female client on warfarin (Coumadin) with an International Normalized Ratio (INR) of 7.5 The client on warfarin (Coumadin) with an INR of 7.5 should be assessed first by the nurse, because this is an elevated result. Normal is considered between 2 and 3. This result is not therapeutic, and the nurse should assess for bleeding and hemodynamic stability. The nurse should report the result to the primary healthcare provider and implement bleeding precautions. The other results are within normal ranges: hemoglobin for a male is 14-18 g/dL; serum calcium is 9.0-10.5 mg/dL; BUN is 5-20 mg/dL and creatinine is 0.7-1.5 mg/dL.

A client who has been admitted to the hospital with chest pain complains of shortness of breath, weakness, and vomiting. The nurse suspects cardiac arrest. Which site is the most appropriate place to check the client's pulse rate?

Femoral A client with chest pain, shortness of breath, weakness, and vomiting may be experiencing cardiac arrest. In a client with cardiac arrest, the most appropriate place to check the pulse rate is the femoral site, because other pulses may not be palpable at this time.

A client with severe breathing problems is brought to the emergency ward. Which level of Maslow's hierarchy of needs does the nurse need to follow?

First

On the second day of hospitalization a client is discussing with the nurse concerns about unhealthy family relationships. During the nurse-client interaction the client begins to talk about a job problem. The nurse's response is: "Let's go back to what we were just talking about." What therapeutic communication technique did the nurse use? 1 Focusing 2 Restating 3 Exploring 4 Accepting

Focusing

A nurse is assigned to change a central line dressing. The agency policy is to clean the site with povidone-iodine and then cleanse with alcohol. The nurse recently attended a conference that presented information that alcohol should precede povidone-iodine in a dressing change. In addition, an article in a nursing journal stated that a new product was a more effective antibacterial than alcohol and povidone-iodine. The nurse has a sample of the new product. How should the nurse proceed?

Follow the agency's policy unless it is contradicted by a primary healthcare provider's prescription

Which factor is used to assess the quality of health care provided to a client?

Functional health status of the client after discharge

The nurse is assessing a client with impaired hearing. Which action of the nurse is most important for establishing a good communication with the client?

Getting the client's attention before speaking

A client is hospitalized with an overdose of benzodiazepines and presents with a respiratory rate less than 10 breaths per minute. Which nursing intervention should be provided as the first priority?

Give oxygen

A 13-month-old toddler has a respiratory tract infection with a low-grade fever. When teaching the parents, which intervention should the nurse emphasize?

Giving small amounts of clear liquids frequently to prevent dehydration

Which type of theory is the Neuman systems model?

Grand theory

Which healthcare system focuses solely on palliative care?

Hospice

During an annual physical assessment a client reports not being able to smell coffee and most foods. Which cranial nerve function should the nurse assess?

I

The registered nurse is evaluating the statements of a new orienting nurse about wound dressing. Which statement made by the new orienting nurse indicates the need for further teaching?

I should take the cotton swap placed on the table

The registered nurse is teaching a student nurse about the interventions to be followed by a client to prevent the spread of infection. Which statement made by the student nurse indicates the need for further learning?

I will advise the client to squeeze pustules

What is a basic concept associated with rehabilitation that the nurse should consider when formulating discharge plans for clients? 1 Rehabilitation needs are met best by the client's family and community resources. 2 Rehabilitation is a specialty area with unique methods for meeting clients' needs. 3 Immediate or potential rehabilitation needs are exhibited by clients with health problems. 4 Clients who are returning to their usual activities following hospitalization do not require rehabilitation.

Immediate or potential rehabilitation needs are exhibited by clients with health problems. Rehabilitation refers to a process that assists clients to obtain optimal functioning. Care should be initiated immediately when a health problem exists to avoid complications and facilitate recuperation. All resources that can be beneficial to client rehabilitation, including the private health care provider and acute care facilities, should be used. Rehabilitation is a commonality in all areas of nursing practice. Rehabilitation is necessary to help clients return to a previous or optimal level of functioning.

Which condition is an example of a bacterial infection?

Impetigo

According to Erikson's theory of psychosocial development, individuals need to accomplish a particular task before effectively grasping this stage and progressing to the next one. What does the Initiative versus Guilt stage include? Select all that apply.

In this stage, an individual pretends and tries out new roles. In this stage, an individual may develop a superego or a conscience. In this stage, an individual fantasizes and imagines discovering the environment

Which of these refers to the accountability element of the decision making process?

Individuals being answerable for their actions

The nurse enters the client's room to do the beginning of shift assessment and notices the client has no pulse. What should be the nurse's first intervention?

Initiating cardiopulmonary resuscitation

A day after an explanation of the effects of surgery to create an ileostomy, a 68-year-old male client remarks to the nurse, "It will be difficult for my wife to care for a helpless old man." This comment by the client regarding himself is an example of Erikson's conflict of: 1 Initiative versus guilt 2 Integrity versus despair 3 Industry versus inferiority 4 Generativity versus stagnation

Integrity versus despair According to Erikson, poor self-concept and feelings of despair are conflicts manifested in those who are older than 65 years of age. The initiative versus guilt conflict is manifested in early childhood between 3 and 6 years of age. The industry versus inferiority conflict is manifested during the ages from 6 to 11 years. The generatively versus stagnation conflict is manifested during middle adulthood, 45 to 65 years of age.

Which of these is a one-on-one communication between a nurse and another person?

Interpersonal communication

Which theorist suggested that the goal of nursing is to use communication to help clients reestablish a positive adaptations to their environments?

King

The goal of a particular nursing theory is to use communication to help a client re-establish positive adaptation to environment, and the framework for the nursing practice is based on treating nursing as a dynamic interpersonal process among the nurse, the client, and the healthcare system. Which nursing theory are these points related to?

King's theory

A nurse is caring for a group of clients with diverse cultural backgrounds. Which nursing theory does the nurse use as a guide?

Leininger's theory

The most effective time to teach clients who have sustained a sudden, traumatic, major loss is most often during the acceptance or adaptation stage of coping. The rationale for this fact is that clients in this stage are: 1 Ready for discharge and therefore in need of preparation 2 At the peak of mental anguish and therefore open to change 3 Less angry and therefore more compliant and more receptive 4 Less anxious and more aware of reality and therefore ready to learn

Less anxious and more aware of reality and therefore ready to learn

A pain scale of 1 to 10 is used by a nurse to assess a client's degree of pain. The client rates the pain as an 8 before receiving an analgesic and a 7 after being medicated. What conclusion should the nurse make regarding the client's response to pain medication?

Medication is not adequately effective

An older adult experiencing delirium suffers from a leg fracture caused by a fall. Which interventions should the nurse follow to prevent future falls?

Minimizing medications Modifying the home environment Manage foot and footwear problems

A registered nurse is teaching a nursing student about Nightingale's theory of nursing. Which statements have been correctly stated by the nursing student as a result of the teaching? Select all that apply

Nightingale's theory states that the focus of nursing is caring through the environment. Nightingale's theory is oriented towards providing fresh air, light, warmth, cleanliness, quiet, and adequate nutrition Nightingale's theory focuses on helping the client deal with the symptoms and changes in function related to an illness.

Which assessment finding in a client signifies a mild form of hypocalcemia?

Numbness around the mouth

In which role does the nurse oversee the budget of a specific nursing unit or agency?

Nurse Manager

A 2-year-old child admitted with a diagnosis of pneumonia was administered antibiotics, fluids, and oxygen. The child's temperature increased until it reached 103° F. When notified, the health care provider determined that there was no need to change treatment, even though the child had a history of febrile seizures. Although concerned, the nurse took no further action. Later, the child had a seizure that resulted in neurological impairment. Legally, who is responsible for the child's injury? 1 Health care provider, because this decision took precedence over the nurse's concern 2 Health care provider, because of total responsibility for the child's health and treatment regimen 3 Nurse, because failure to further question the health care provider about the child's status placed the child at risk 4 Neither, because high fevers are common in children and the health care provider had little cause for concern

Nurse, because failure to further question the health care provider about the child's status placed the child at risk It is the nurse's responsibility to foresee potential harm and prevent risks by acting as a client advocate. This is not acceptable as a rationale for inaction. The nurse and health care provider share interdependent roles in the assessment and care of clients. High temperatures are common in children but are nonetheless a valid cause for concern.

What are the priority nursing interventions for a client with neutropenia in an emergency department?

Obtain blood cultures immediately, Administer antibiotics STAT as prescribed

A nurse is educating a client about the importance of self-care. Which nursing theory lays the inspiration for this?

Orem's Theory

While teaching a diabetic client about self-care management, the nurse asks the client to inspect his or her feet daily and to wear well-fitted shoes to prevent foot ulcers. Which nursing theory is applicable in this situation?

Orem's theory

Which client assessment finding should the nurse document as subjective data? 1 B/P 120/82 2 Pain rating of five (5) 3 Potassium 4.0 mEq 4 Pulse oximetry reading of 96%

Pain rating of five (5)

Which group benefits from Medicare?

People who are 65 years or older

Which theory focuses on developing the interpersonal relationships between the nurse, client, and the client's family?

Peplau's theory

Which of these is a part of health belief model?

Perception of susceptibility to an illness

After surgery for a fractured hip, a client states, "I don't remember when I have ever been so uncomfortable." What should the nurse's initial response be?

Perform a complete pain assessment

Which nursing intervention for opening the airway should be performed in an unconscious client with a spinal injury?

Performing a jaw thrust maneuver. The jaw thrust maneuver is the recommended procedure for opening the airway of an unconscious client with a possible spinal or neck injury. Needle thoracostomy should be performed in a client with absent breath sounds. Cardiopulmonary resuscitation should be initiated in a client when there is no pulse. Providing oxygen via a nonrebreather mask is mainly performed when the client is conscious.

The registered nurse is assisting a client who is hospitalized with high fever. Which task delegated to the unlicensed assistive personnel (UAP) would be appropriate?

Performing all hygiene tasks Helping the client in changing clothes

A 90-year-old female resident of a nursing home falls and fractures the proximal end of her right femur. The surgeon plans to reduce the fracture with an internal fixation device. The general fact about the older adult that the nurse should consider when caring for this client is that: 1 Aging causes a lower pain threshold 2 Physiological coping defenses are reduced 3 Most confused states result from dementia 4 Older adults psychologically tolerate changes well

Physiological coping defenses are reduced

Creaking or grating sounds caused by roughened, inflamed pleural surfaces rubbing together. They are associated with pleurisy, pneumonia, or a pulmonary infarct.

Pleural friction rubs

A nurse notices that a child is playing with a doll and is practicing hygiene by brushing its hair and teeth. In which stage of Piaget's theory of cognitive development should the nurse expect this child to be considered?

Preoperational stage

Which key feature does the nurse associate with a stage 2 pressure ulcer?

Presence of nonintact skin

What is the primary focus of the nurse when providing evidence-based care to the client?

Problem-solving approach

In which of these activities does a nurse engage when providing tertiary care at a rehabilitation center? Select all that apply.

Providing work therapy for hospitalized patients Making provisions to help clients maximize their capabilities

How is public health nursing different from community health nursing?

Public health nursing focuses on population

The nurse should place the client in which position to obtain the most accurate reading of jugular vein distention? 1 Upright at 90 degrees 2 Supine position 3 Raised to 45 degrees 4 Raised to 10 degrees

Raised to 45 degrees

A primary healthcare provider writes a prescription of "Restraints PRN" for a client who has a history of violent behavior. What is the nurse's responsibility in regard to this prescription?

Recognizing that PRN prescriptions for restraints are unacceptable

A hospice client who has severe pain asks for another dose of oxycodone. What is the nurse's primary consideration when responding to the client's request?

Reduce the client's pain

The nurse finds that the client's fever spikes and falls without a return to a normal level. Which pattern of fever is this a characteristic of?

Remittent

Which intervention would be most beneficial in preventing a catheter-associated urinary tract infection in a postoperative client?

Removing the catheter within 24 hours

Which nursing action is confidential and protected from legal action? 1 Providing health teaching regarding family planning. 2 Offering first aid at the scene of an automobile collision. 3 Reporting incidents of suspected child abuse to the appropriate authorities. 4 Administering resuscitative measures to an unconscious child pulled from a swimming pool

Reporting incidents of suspected child abuse to the appropriate authorities

A client receives a prescription for morphine via patient-controlled analgesia (PCA). Before beginning administration of this medication, what should the nurse assess first?

Respirations

A client is receiving oxycodone postoperatively for pain. The healthcare provider's prescription indicates that the dose should be administered every 3 hours for eight doses. What should the nurse assess before administering each dose of oxycodone?

Respiratory rate and level of consciousness

The nurse is caring for a client with chronic pain who is on opioid treatment. The client has constipation, nausea, vomiting, level 3 sedation, respiratory rate of 8 breaths per minute, and pruritus. Which conditions of the client should the nurse consider as highest priority?

Respiratory rate and sedation. Level 3 sedation needs immediate intervention

A pregnant client's history reveals opioid abuse. What is the nurse's initial plan for providing pain relief measures during labor?

Scheduling pain medication at regular intervals

The nurse is caring for a client with a platelet count of 50,000 cells per microliter. Which recommendation is inappropriate for the client?

Shaving with a straight blade

What should the nurse teach a client who is taking antihypertensives to do to minimize orthostatic hypotension?

Sit on the edge of the bed for 5 minutes before standing

A client with Crohn disease is admitted to the hospital with abdominal pain, fever, poor skin turgor, and diarrhea, with 10 stools in the past 24 hours. Which signs are evidence that the client most likely is dehydrated?

Sunken eyes Dry mucous membranes

Which statement made by a nursing student about Swanson's theory of caring needs correction?

Swanson's theory of caring provides a basis to help nurses understand how clients cope with uncertainty and the illness response.

A nursing student is recalling the hierarchy of evidence available for conducting research. In which order should the nurse give priority to the findings of a study?

Systematic reviews and meta-analysis of randomized controlled trials (RCTs) One well designed randomized controlled trial (RCT) Well-designed controlled trials without randomization Well-designed case control studies Systematic reviews of descriptive and qualitative studies Single descriptive or qualitative studies Opinions of authorities or expert committees https://eaq.elsevier.com/Libraries/EAQ_NCLEX-RN_2-0/QB-k3dz/Q-zr95-g81l9yki/g11e/Potter_Figure_5-2.png

Which statement is true of the middle childhood stage according to Kohlberg's development of moral reasoning? Select all that apply.

The child is in the conventional level The child develops a good boy-nice girl orientation

A middle-aged client says, "I have been unsuccessful in raising my kids." Which stage should the nurse expect the client to have reached, according to Erikson's theory of psychosocial development?

The client has reached the Generativity versus Self-Absorption and Stagnation stage.

An elderly adult with Parkinson's disease falls while going to the bathroom and gets injured. The nurse taking care of the client informs the primary healthcare provider. What step should the nurse take to alert the risk management system?

The nurse should document the incident in the occurrence report tool

Which of the following nursing interventions promotes perfusion and healing of the surgical wound for an older adult?

The nurse should keep the client adequately hydrated

A 3-year-old child with eczema of the face and arms has disregarded the nurse's warnings to "stop scratching, or else!" The nurse finds the toddler scratching so intensely that the arms are bleeding. The nurse then ties the toddler's arms to the crib sides, saying, "I'm going to teach you one way or another." How should the nurse's behavior be interpreted? 1.These actions can be construed as assault and battery. 2.The problem was resolved with forethought and accountability. 3.Skin must be protected, and the actions taken were by a reasonably prudent nurse. 4.The nurse had tried to reason with the toddler and expected understanding and cooperation

These actions can be construed as assault and battery Assault is a threat or an attempt to do violence to another, and battery means touching an individual in an offensive manner or actually injuring another person.

What is the purpose of evaluation research?

To test how well a program, practice, or policy is working

The nurse providing care for a client with a diagnosis of neutropenia reviews isolation procedures with the client's spouse. The nurse determines that the teaching was effective when the spouse states that protective environment isolation helps prevent the spread of infection: 1 To the client from outside sources. 2 From the client to others. 3 From the client by using special techniques to destroy infectious fluids and secretions. 4 To the client by using special sterilization techniques for linens and personal items.

To the client from outside sources

Which assessment finding is considered the earliest sign of decreased tissue oxygenation?

Unexplained restlessness

While reviewing the medical reports in an acute care setting, the nurse finds that the client is at risk for kidney damage and requests the healthcare provider to increase the intravenous fluid rate as a priority nursing intervention. Which finding supports the nurse's conclusion?

Urine output is 25mL/hr

The registered nurse finds that a client cared for by a student nurse has developed an infection. Which action of the student nurse does the registered nurse suspect to be the cause of infection?

Use of a wet dressing

A nurse is caring for a client with acquired immunodeficiency syndrome (AIDS). What precautions should the nurse take when caring for this client?

Use standard precautions

Normal, low-pitched rustling sounds heard over peripheral lung fields

Vesicular breath sounds

A nurse is assessing a client who underwent abdominal surgery 10 days ago. The client complains of pain in the abdomen. What type of pain does the client experience?

Visceral pain

A client in the final stage of cancer is very anxious about his or her disease. The client is showing aggressive behavior towards family members. The nurse comforts and offers compassion and empathy to the client and the family members. Which theory principle is the nurse following in this situation?

Watson's Theory

Continuous high-pitched squeaking or musical sounds that result from rapid vibration of bronchial walls. They are associated with bronchospasms or airway obstruction

Wheezing

A client who has a long leg cast for a fractured bone is to be discharged from the emergency department. When discussing pain management, when does the nurse advise the client to take the prescribed as-needed oxycodone?

When the discomfort begins

A nurse is caring for an older adult who is taking acetaminophen for the relief of chronic pain. Which substance is most important for the nurse to determine if the client is taking because it intensifies the most serious adverse effect of acetaminophen?

alcohol

What is the priority nursing action for a client with delirium?

creating a calm safe environment

After abdominal surgery a client reports pain. What action should the nurse take first?

determine the characteristics

What clinical finding does a nurse anticipate when admitting a client with an extracellular fluid volume excess?

distended jugular veins

The nurse reviews a medical record and is concerned that the client may develop hyperkalemia. Which disease increases the risk of hyperkalemia?

end stage renal failure

Arrange in order the items of personal protection equipment (PPE) removed after performing a surgical procedure.

glove, face shield, gown, mask

A client has been admitted with a diagnosis of intractable vomiting and can only tolerate sips of water. The initial blood work shows a sodium level of 122 mEq/L (122 mmol/L) and a potassium level of 3.6 mEq/L (3.6 mmol/L). Based on the lab results and symptoms, what is the client experiencing?

hyponatremia

A client reports vomiting and diarrhea for 3 days. Which clinical indicator is most commonly used to determine whether the client has a fluid deficit?

loss of body weight

After assessing a client's vision, the nurse observes the client has difficulty seeing objects that are far away. Which condition does the nurse suspect the client is experiencing?

myopia

A client reports severe pain 2 days after surgery. After assessing the characteristics of the pain, which initial action should the nurse take next?

obtain vitals

Arrange the hierarchy of needs in ascending order beginning with the highest priority needs as defined by Maslow. Correct 1. Physiological needs Correct 2. Safety and security Correct 3. Love and belonging needs Correct 4. Self-esteem Correct 5. Self-actualization

perfect

The nurse must understand the process of changing behaviors to be able to support difficult behavioral changes in clients. Arrange the Stages of Health Behavior Change as described by DiClemente and Prochaska (1998) in the transtheoretical model of change. Correct 1. Precontemplation Correct 2. Contemplation Correct 3. Preparation Correct 4. Action Correct 5. Maintenance stage

perfect

When monitoring fluids and electrolytes, the nurse recalls that the major cation-regulating intracellular osmolarity is what?

potassium

Which type of debridement is most often used to quickly remove large amounts of a client's nonviable tissue?

surgical

A client complains of pain. Which question asked by the nurse are most appropriate to assess the nature of the pain?

"Can you describe your pain to me?"

A client shows an increase in rate respirations that are abnormally deep and regular. What condition would the nurse expect? 1 Hypoventilation 2 Biot's respiration 3 Kussmaul's respiration 4 Cheyne-Stokes respiration

3

A registered nurse is teaching a nursing student about Maslow's hierarchy of needs. Which statement made by the nursing student needs correction

"The fourth level contains love and belonging needs, including friendship, social relationships, and sexual love.

A registered nurse is teaching a nursing student about Maslow's hierarchy of needs. Which statement made by the nursing student needs correction?

"The fourth level contains love and belonging needs, including friendship, social relationships, and sexual love."

The nurse teaches a nursing student about the discharge instructions to be given to a post-operative client. Which statement made by the nursing student indicates the nurse needs to intervene?

"you should use a saline solution on non oozing wounds"

The nurse assessed a client's pulse rate and recorded the score as 3+. What is the strength of the pulse? 1 Strong 2 Bounding 3 Expected 4 Diminished

1

Immediately after receiving spinal anesthesia a client develops hypotension. To what physiological change does the nurse attribute the decreased blood pressure? 1 Dilation of blood vessels 2 Decreased response of chemoreceptors 3 Decreased strength of cardiac contractions 4 Disruption of cardiac accelerator pathways

1 Dilation of blood vessels Paralysis of the sympathetic vasomotor nerves after administration of a spinal anesthetic results in dilation of blood vessels, which causes a subsequent decrease in blood pressure.

The nurse is discussing discharge plans with a client who had a myocardial infarction. The client states, "I'm worried about going home." The nurse responds, "Tell me more about this." What interviewing technique did the nurse use? 1 Exploring 2 Reflecting 3 Refocusing 4 Acknowledging

1 Exploring

A nurse is caring for an older adult with a hearing loss secondary to aging. What can the nurse expect to identify when assessing this client? (Select all that apply.) 1 Dry cerumen 2 Tears in the tympanic membrane 3 Difficulty hearing high-pitched voices 4 Decrease of hair in the auditory canal 5 Overgrowth of the epithelial auditory lining

1 Dry cerumen 3 Difficulty hearing high-pitched voices Cerumen (ear wax) becomes drier and harder as a person ages. Generally, female voices have a higher pitch than male voices; older adults with presbycusis (hearing loss caused by the aging process) have more difficulty hearing higher-pitched sounds. There is no greater incidence of tympanic tears caused by the aging process. The hair in the auditory canal increases, not decreases. The epithelium of the lining of the ear becomes thinner and drier

When suctioning a client with a tracheostomy, an important safety measure for the nurse is to: 1 Hyperventilate the client with room air prior to suctioning. 2 Apply suction only as the catheter is being withdrawn. 3 Insert the catheter until the cough reflex is stimulated. 4 Remove the inner cannula before inserting the suction catheter

2 Apply suction only as the catheter is being withdrawn. Use of suction upon withdrawal of a suction catheter reduces unnecessary removal of oxygen. In addition, suction should be applied intermittently during the withdrawal procedure to prevent hypoxia. A sterile catheter is used to prevent infection, and the catheter should only be inserted approximately 1 to 2 cm past the end of the trach tube to prevent tissue trauma. Hyperventilating a client before suctioning should always be with oxygen, not room air. Inserting the catheter until the cough reflex is stimulated frequently occurs and does help to mobilize secretions but is not a safety measure. Removal of the inner cannula before inserting the suction catheter is not necessary. Topics

The nurse is caring for a client with a closed soft tissue injury. The nurse describes the injury as a/an: 1 Abrasion 2 Contusion 3 Laceration 4 Avulsion

2 Contusion Closed wounds are considered contusions and hematomas because the skin is not broken. Abrasions, lacerations, and avulsions are considered open because there is a break in the skin integrity.

While undergoing a soapsuds enema, the client reports abdominal cramping. What action should the nurse take? 1 Immediately stop the infusion. 2 Lower the height of the enema bag. 3 Advance the enema tubing 2 to 3 inches. 4 Clamp the tube for 2 minutes, then restart the infusion

2 Lower the height of the enema bag. Abdominal cramping during a soapsuds enema may be due to too rapid administration of the enema solution. Lowering the height of the enema bag slows the flow and allows the bowel time to adapt to the distention without causing excessive discomfort. Stopping the infusion is not necessary. Advancing the enema tubing is not appropriate. Clamping the tube for several minutes and then restarting the infusion may be attempted if slowing the infusion does not relieve the cramps.

A client has a stage III pressure ulcer. Which nursing intervention can prevent further injury by eliminating shearing force? 1 Maintain the head of the bed at 35 degrees or less. 2 With the help of another staff member, use a drawsheet when lifting the client in bed. 3 Reposition the client at least every 2 hours and support the client with pillows. 4 At least once every 8 hours, perform passive range-of-motion exercises of all extremities

2 With the help of another staff member, use a drawsheet when lifting the client in bed.

Which nursing interventions require a nurse to wear gloves? (Select all that apply.) 1 Giving a back rub. 2 Cleaning a newborn immediately after delivery. 3 Emptying a portable wound drainage system. 4 Interviewing a client in the emergency department. 5 Obtaining the blood pressure of a client who is human immunodeficiency virus (HIV) positive

2 Cleaning a newborn immediately after delivery. 3 Emptying a portable wound drainage system.

A nurse is hired to work in a health care facility that has a complete computer-based client information system. The nurse in charge knows that the newly hired nurse is knowledgeable about this system when the nurse says: 1. "More medication errors are made when this system is used." 2. "It is disappointing that nurses are not allowed to use this system." 3. "Client information is immediately available when this system is used." 4. "I will have less time to provide direct care to my clients with this system."

3 "Client information is immediately available when this system is used."

A client who is dying appears happy and tells a nurse a joke about the situation despite becoming sicker and weaker. What is the nurse's most therapeutic response? 1 "Why are you always telling jokes?" 2 "Your laughter is a cover for your fear." 3 "Does it help to joke about your illness?" 4 "The one who laughs on the outside cries on the inside."

3 "Does it help to joke about your illness?"

A nurse identifies that an older adult has not achieved the desired outcome from a prescribed proprietary medication. When assessing the situation, the client shares that the medication is too expensive and the prescription was never filled. What is an appropriate nursing response? 1 Ask the pharmacist to provide a generic form of the medication. 2 Encourage the client to acquire the medication over the internet. 3 Inform the health care provider of the inability to afford the medication. 4 Suggest that the client purchase insurance that covers prescription medications.

3 Inform the health care provider of the inability to afford the medication.

A nurse fails to act in a reasonable, prudent manner. Which legal principle is most likely to be applied? 1 Malice 2 Tort law 3 Malpractice 4 Case law

3 Malpractice

A nurse is discussing Alcoholics Anonymous (AA) with a client. What behavior expected of members of AA should the nurse include in the discussion? 1 Speaking aloud at weekly meetings 2 Promising to attend at least 12 meetings yearly 3 Maintaining controlled drinking after six months 4 Acknowledging an inability to control the problem

4 Acknowledging an inability to control the problem

A nurse takes into consideration that the key factor in accurately assessing how a client will cope with body image changes is the: 1 Suddenness of the change 2 Obviousness of the change 3 Extent of the body changes 4 Perception of the body changes

4 Perception of the body changes

An 82-year-old retired schoolteacher is admitted to a nursing home. During the physical assessment, the nurse identifies an ocular problem common to persons at this client's developmental level, which is: 1 Tropia 2 Myopia 3 Hyperopia 4 Presbyopia

4 Presbyopia Presbyopia is the decreased accommodative ability of the lens that occurs with aging. Tropia (eye turn) generally occurs at birth. Myopia (nearsightedness) can occur during any developmental level or be congenital. Hyperopia (farsightedness) can occur during any developmental level or be congenital.

The normal RBC count for a healthy male

4.7-6.1 million

A registered nurse is teaching a nursing student about Piaget's theory of cognitive development that includes four periods, which are related to age. Which age group corresponds with concrete operations?

7- 11 years

A nursing student is recording the radial pulse rate in a client with dysrhythmias and documented a radial pulse of 80 beats per minute. The registered nurse reassesses the client and notices a pulse deficit of 15. What is the client's apical pulse?

95

A nurse suspects that a client has poison ivy. Assessment findings reveal vesicles on the arms and legs. A vesicle can be described as: 1 A lesion filled with purulent drainage. 2 An erosion into the dermis. 3 A solid mass of fibrous tissue. 4 A lesion filled with serous fluid.

A lesion filled with serous fluid. A vesicle is a small blisterlike elevation on the skin containing serous fluid. Vesicles are usually transparent. Common causes of vesicles include herpes, herpes zoster, and dermatitis associated with poison oak or ivy. A lesion filled with purulent drainage is known as a pustule; an erosion into the dermis is known as an excoriation or ulcer; and a solid mass of fibrous tissue is known as a papule.

A hospitalized client experiences a fall after climbing over the bed's side rails. Upon reviewing the client's medical record, the nurse discovers that restraints had been prescribed but were not in place at the time of the fall. What information should the nurse include in the follow-up incident report?

A listing of facts related to the incident as witnessed by the nurse

A registered nurse is educating a nursing student on the various classifications of torts. What acts are classified as intentional torts in nursing practice? Select all that apply. A. Battery B. Assault C. Negligence D. Malpractice E. False Imprisonment

A, B, and E. Intentional torts include battery, assault, and false imprisonment. Unintentional torts include negligence and malpractice.

A client is transferred to an acute care nursing unit after surgery. Which action of the nurse is most important and should be performed first?

Assess the patency of airway.

Which theory is based on the model of primacy of caring?

Benner and Wrubel's Theory

How does a nurse prepare a "factual" record when performing a client documentation?

By recording descriptive and objective information of what the nurse sees, hears, feels, and smells

A nurse is caring for a client with pain after surgery. The nurse takes the blood pressure and pulse rate of the client and asks the client to rate the level of pain on the pain scale. The nurse then notifies the primary healthcare provider. Which standard of practice does the nurse perform? A. Planning B. Diagnosis C. Assessment D. Implementation

C. Assessment When a nurse collects comprehensive data relevant to the client's health or the situation, it is considered assessment. In the given scenario, the nurse is assessing the client to minimize pain. Planning refers to instances when a nurse develops a plan to attain expected outcomes. Diagnosis refers to instances when the nurse analyzes the assessment data to determine the diagnoses or issues. Implementation refers to instances when the nurse implements the identified plan.

A nurse applies a cold pack to treat an acute musculoskeletal injury. Cold therapy decreases pain by doing what?

Causing local vasoconstriction, preventing edema and muscle spasms

What condition would a nurse suspect in a client with abnormal respirations with alternating periods of apnea and rapid breathing?

Cheyne-Stokes respirations

For which illness should airborne precautions be implemented?

Chickenpox

A nurse is caring for a newly admitted client in a long-term care facility. The nurse notes that the client has a decreased attention span and cannot concentrate. The nurse suspects which effects of sensory deprivation?

Cognitive response

A 65-year-old client tells the nurse, "I see some particles that float within my field of vision." What may be the cause of this condition?

Liquefaction and detachment of the vitreous membrane

After several weeks of caring for clients who are in the terminal stage of illness, the nurse becomes aware of feeling depressed when coming to work. What should the nurse do? 1 Talk with other nurses on the unit. 2 Take several personal days off from work. 3 Limit emotional involvement with the clients. 4 Request a transfer to another area of the hospital.

Correct1 Talk with other nurses on the unit. Talking with nurses who cope with similar issues allows the nurse to share feelings and obtain constructive emotional support. Avoidance may provide an immediate solution, but it works only for a short time. The nurse will eventually have to work through feelings. Limiting emotional involvement with the clients avoids personal feelings about death and dying and is an unacceptable attitude when caring for dying clients. Emotional withdrawal may be perceived by the clients as rejection. Avoidance may provide an immediate solution, but it works only for a short time. The nurse will eventually have to work through feelings.

The nurse is providing restraint education to a group of nursing students. The nurse should include that it is inappropriate to use a restraint device to: 1 Prevent a client from pulling out an IV when there is concern that the client cannot follow instructions or is confused. 2 Prevent an adult client from getting up at night when there is insufficient staffing on the unit. 3 Maintain immobilization of a client's leg to prevent dislodging a skin graft. 4 Keep an older adult client from falling out of bed following a surgical procedure.

Correct2 Prevent an adult client from getting up at night when there is insufficient staffing on the unit.

The nurse performs a respiratory assessment and auscultates breath sounds that are high-pitched, creaking and accentuated on expiration. Which term best describes the findings? 1 Rhonchi 2 Wheezes 3 Pleural friction rub 4 Bronchovesicular

Correct2 Wheezes Wheezes are one of the most common breath sounds assessed and auscultated in clients with asthma and COPD. Wheezes are produced as air flows through narrowed passageways. Rhonchi are coarse rattling sounds similar to snoring and are usually caused by secretions in the bronchial airways. A pleural friction rub is an abrasive sound made by two acutely inflamed serous surfaces rubbing together during the respiratory cycle. Bronchovesicular sounds are intermediate between bronchial (upper) and vesicular (lower) breath sounds; they are normal when heard between the first and second intercostal spaces anteriorly and posteriorly between scapulae.

A visitor says to the nurse, "Can I read my client's progress record? I am the sponsor from an alcohol recovery program." How should the nurse respond? 1.Allow the visitor to review the record; sponsors have access to privileged information 2.Ask the primary health care provider about granting permission to the sponsor 3.Do not allow the sponsor to review the record 4.Allow the visitor to review the record; clients with alcoholism need reassurance from sponsors

Do not allow the sponsor to review the record

Which degree does The American Association of Colleges of Nursing (AACN) recommend as terminal practice degree for all advanced practice registered nurses (APRNs)?

Doctor of Nursing Practice (DNP)

When caring for a client with pneumonia, which nursing intervention is the highest priority? 1 Increase fluid intake. 2 Employ breathing exercises and controlled coughing. 3 Ambulate as much as possible. 4 Maintain an NPO status.

Employ breathing exercises and controlled coughing. For most clients, the most effective means of preventing fluid consolidation in the lungs with a diagnosis of pneumonia is to keep active by deep breathing and controlled coughing exercises. Increased fluid intake and ambulation are important aspects of care if not contraindicated, but they are secondary to deep breathing and coughing. Keeping the client NPO is not necessary; unless contraindicated, the client with pneumonia is usually offered his or her regular diet as tolerated.

While assessing an older adult, the nurse observes visual impairment in the client. Which technique should the nurse use to communicate?

Encourage the older adult to use assistive devices such as glasses

A client expresses concern about the surgical consent that the client signed. How should the nurse respond? 1 Share the client's concern with the family 2 Inform the health care provider of the client's concern 3 Reassure the client that the surgery will be successful 4 Cancel the surgery until the client feels more comfortable with the decision

Inform the health care provider of the client's concern

A client who had thoracic surgery complains of pain at the incision site when coughing and deep breathing. What action should the nurse take?

Instruct the client to splint the wound with a pillow when coughing.

According to Erikson's theory of psychosocial development, which opposing conflicts is an older adult likely to face?

Integrity versus Despair

A client who is legally blind is admitted to the hospital for surgery. Which nursing action is most appropriate when caring for this client?

Keep the furniture in the same location in the room

A nursing student notes the characteristics of middle-range theories. Which points noted by the nursing student are accurate? Select all that apply.

Middle-range theories provide a basis to help nurses understand how clients cope with uncertainty and the illness response Middle-range theories include Mishel's theory of uncertainty in illness, which focuses on a client's experiences with cancer while living with continual uncertainty Middle-range theories tend to focus on a specific field of nursing (such as uncertainty, incontinence, social support, quality of life, and caring) rather than reflect on a wide variety of nursing care situations.

A nurse has provided discharge instructions to a client who received a prescription for a walker to use for assistance with ambulation. The nurse determines that the teaching has been effective when the client: 1. Picks up the walker and carries it for short distances. 2. Uses the walker only when someone else is present. 3. Moves the walker no more than 12 inches in front of the client during use. 4. States that a walker will be purchased on the way home from the hospital

Moves the walker no more than 12 inches in front of the client during use

Which of these records can a nurse use to document information specific to the client's health in a story-like format?

Narrative documentation

After an above-the-knee amputation of a right leg, a client reports pain in the right foot. The nurse should inform the client that phantom limb pain is the result of what?

Nerve endings in the limb that are still intact and react to stimuli

Which theory emphasizes the facilitation of the body's reparative processes by manipulating the client's environment?

Nightingale's theory

Which points about nursing care and nursing practice have been accurately stated? Select all that apply.

Nursing theories help to describe, explain, predict, and/or prescribe nursing care measures. The scientific work used in developing theories expands the scientific knowledge of the profession The expertise required to interpret clinical situations and make clinical judgments is the essence of nursing care and the basis for advancing nursing practice and nursing science.

Which nursing theory focuses on the client's self-care needs?

Orem's theory

Which physical assessment findings of a client suspected of having a respiratory disorder would be considered normal? Select all that apply.

Pink nasal mucosa, midline trachea, non labored breathing of 14/min

Which theory details nursing interventions for a specific phenomenon and the expected outcome of care?

Prescriptive theories

When a nurse requests that a client's pain intensity be rated on a scale of 0 to 10, the client states that the pain is "99." How does the nurse interpret the client's behavior?

Requires an intervention immediately

A nurse is teaching a parent about the different temperaments that a child may display. What characteristics does a slow-to-warm up child display? Select all that apply.

The child adapts slowly with frequent communication. The child reacts with mild but passive resistance to novelty. The child reacts negatively and with mild intensity to new stimuli.

What is the correct order of steps of the nursing diagnostic process? Incorrect 1. Identify the client's needs. Incorrect 2. Assess the client's health status. Incorrect 3. Look for defining characteristics. Correct 4. Cluster data. Incorrect 5. Interpret the meaning of the data. Incorrect 6. Formulate nursing diagnoses. Incorrect 7. Validate the data with other sources.

The diagnostic reasoning process involves the use of assessment data for the client. The assessment data is obtained from the client, family, and health care resources. The nurse validates and ensures the data is accurate and uses critical thinking to interpret and analyze the data before it is classified and organized into data clusters. This organization helps the nurse identify the client's health needs. The nurse then formulates the nursing diagnoses using standard formal nursing diagnostic statements.

What is the primary focus of nursing care in the "family as context" approach?

The health and development of an individual

When should the nurse use hypoallergenic tape or Montgomery straps as the best practice in postoperative skin care?

When protecting the fragile skin of the older client

Arrange the order of steps involved in the evidence-based practice process. 1. Ask a clinical question. 2. Collect the most relevant and best evidence. 3. Critically appraise the evidence you gather. 4. Integrate all evidence with one's clinical expertise and client preferences and values in making a practice decision or change. 5. Evaluate the practice decision or change. 6. Share the outcomes of evidence-based practice.

perfect

What is the correct order of phases a client experiences in the event of a change in body image following an illness? Correct 1. Shock Correct 2. Withdrawal Correct 3. Acknowledgement Correct 4. Acceptance Correct 5. Rehabilitation

perfect

A nurse is evaluating a client's response to fluid replacement therapy. Which clinical finding indicates successful replacement?

urinary output of 30 ml in an hour

Which condition will the nurse monitor for in a client with interruption of venous return?

varicosity

A nurse in a long-term health care setting will introduce a client who has a PhD to the other clients. The client tells the nurse, "I wish to be called Doctor." How should the nurse respond? 1 "Your wish will be respected." 2 "Why do you want to be called Doctor?" 3 "Residents here call one another by their first names." 4 "Wouldn't it be better if the others do not know you are a doctor?"

"Your wish will be respected."

The registered nurse is teaching a nursing student about interventions that reduce the risk of pressure ulcers in a client. Which statements made by the nursing student indicate effective learning? Select all that apply.

- "I will elevate the head of the client's bed to 30 degrees." - "I will instruct the client to take baths in lukewarm water." - "I will ensure that the client's fluid intake is 2000 to 3000 mL/day."

What are the roles of an unlicensed assistive personnel in skin care? Select all that apply.

- To assist the client in bathing - To apply wet dressings to the skin - To report changes in the skin appearance

A child who reports shortness of breath, wheezing, and coughing is found to have pulmonary edema and is prescribed furosemide. Which nursing interventions would be beneficial to the client? Select all that apply.

- checking the child's weight everyday - calculating the dose of drug as carefully as possible - assessing the child regularly to help prevent electrolyte loss

What is a nurse's responsibility when administering prescribed opioid analgesics? Select all that apply.

- count the client's respirations - document the intensity of the client's pain - verify the number of doses in the locked cabinet before administering the prescribed dose

A nurse is aware that an older adult is at risk for social isolation as a result of which factors? Select all that apply.

- experiencing progressive macular degeneration - living alone since a spouse's death 3 years ago

A nurse is caring for a client who sustained skin injuries 3 days ago. Which changes can be noticed by the nurse in the client? Select all that apply.

- local edema - erythema

What are the desired outcomes that the nurse expects when administering a nonsteroidal antiinflammatory drug (NSAID)? Select all that apply.

- pain relief - antipyresis - reduced inflammation

A client receiving morphine is being monitored by the nurse for signs and symptoms of overdose. Which clinical findings support a conclusion of overdose? Select all that apply.

- slow respirations - bradycardia - sedation

A client has received instructions to take 650 mg aspirin every 6 hours as needed for arthritic pain. What should the nurse include in the client's medication teaching? Select all that apply.

- take the aspirin with meals or a snake - do not chew enteric coated tablets - report persistent abdominal pain

The nurse is teaching a group of student nurses about caring for a hospitalized immunosuppressed client. Which statement(s) made by the student nurse indicates the need for further teaching?

-"I should inspect the client's mouth at least every 4 hours." -"I should monitor the vital signs of the client every 8 hours. -"I should ensure that that the client's room and bathroom are cleaned at least once a week."

Elbow restraints have been prescribed for a confused client to keep the client from pulling out a nasogastric tube and indwelling urinary retention catheter. What is most important for the nurse to do

-Assess the client's condition every hour. A restraint impedes the movement of a client; therefore a client's condition needs to be assessed every hour. All restraints are required to be represcribed every 24 hours. Restraints should be removed and activity and skin care provided at least every 2 hours to prevent contractures and skin breakdown. Output from tubes may be monitored hourly, but generally does not need to be documented as frequently as every 2 hours. Generally output from tubes is emptied, measured, and documented at the end of each shift. A client who is in critical condition or in the immediate postoperative period may have urinary output measured hourly because this reflects cardiovascular status

The primary healthcare provider prescribes thioridazine and assigns the nurse to assess the client for orthostatic hypotension. Which interventions would the nurse perform?

-Measuring the blood pressure before dosing -Measuring the blood pressure one hour after dosing -Measuring the blood pressure one or two minutes after the client sits or stands

A client complains to the nurse manager about a coworker. The nurse manager listens to both the patient's and the coworker's side of the story. Which critical thinking quality is shown in this situation? 1 Fairness 2 Discipline 3 Risk-taking 4 Responsibility

1

A client has corrective surgery for a bladder laceration. What nursing intervention takes priority during this client's postoperative period? 1 Turning frequently 2 Raising side rails on the bed 3 Providing range-of-motion exercises 4 Massaging the back three times a day

1

A client is admitted to the hospital with a tentative diagnosis of infectious pulmonary tuberculosis. What infection control measures should the nurse take? 1 Don an N95 respirator mask before entering the room. 2 Put on a permeable gown each time before entering the room. 3 Implement contact precautions and post appropriate signage. 4 After finishing with patient care, remove the gown first and then remove the gloves.

1

A nurse in the ambulatory preoperative unit identifies that a client is more anxious than most clients. What is the nurse's best intervention? 1 Attempt to identify the client's concerns. 2 Reassure the client that the surgery is routine. 3 Report the client's anxiety to the healthcare provider. 4 Provide privacy by pulling the curtain around the client.

1

A nurse is assessing a client who underwent abdominal surgery 10 days ago. The client complains of pain in the abdomen. What type of pain does the client experience? 1 Visceral pain 2 Somatic pain 3 Referred pain 4 Intractable pain

1

The client reports difficulty in breathing. The nurse auscultates lung sounds and assesses the respiratory rate. What is the purpose of the nurse's action? 1 Data collection 2 Data validation 3 Data clustering 4 Data interpretation

1

The nurse assesses an edematous client and recalls that edema occurs in what extracellular fluid compartment? 1 Interstitial 2 Intercellular 3 Intravascular 4 Intracellular

1

The nurse at the well baby clinic is assessing the gross motor skills of a five-month-old infant. Which finding is a cause for concern? 1 The baby has a head lag when pulled to sit. 2 The baby can turn from the side to the back. 3 The baby can turn from the abdomen to the back. 4 The baby supports much of his own weight when he or she is pulled to stand.

1

What is the most important nursing action involved in caring for a client using medications to manage disease? 1 Administering the medications 2 Teaching about the medications 3 Ensuring adherence to the medication regimen 4 Evaluating the client's ability to self-administer medications

1

When assessing a client's fluid and electrolyte status, the nurse recalls that the regulator of extracellular osmolarity is what? 1 Sodium 2 Potassium 3 Chloride 4 Calcium

1

Which assessment finding of the skin refers to elasticity? 1 Turgor 2 Edema 3 Texture 4 Vascularity

1

Which nursing action would be considered a part of self-regulation in the decision-making process? 1 Reflecting on one's own experiences 2 Looking at all the situations objectively 3 Supporting findings and conclusions 4 Making careful assumptions about a client's information

1

When caring for a client with a fractured hip, the nurse should place pillows around the injured leg to specifically maintain: 1 Abduction. 2 Adduction. 3 Traction. 4 Elevation

1 Abduction.

Health promotion efforts with the chronically ill client should include interventions related to primary prevention. What should this include? 1 Encouraging daily physical exercise 2 Performing yearly physical examinations 3 Providing hypertension screening programs 4 Teaching a person with diabetes how to prevent complications

1 Encouraging daily physical exercise

A nurse is assessing an older adult client. Which clinical findings are expected responses to the aging process? Select all that apply. 1 Slowed neurologic responses 2 Lowered intelligence quotient 3 Long-term memory impairment 4 Forgetfulness about recent events 5 Reduced ability to maintain an erection

1,4,5

A nurse is taking the vital signs of a client who has just been admitted to the healthcare facility. Which intervention by the nurse provides greater client satisfaction? 1 The nurse records the vital signs and leaves the room. 2 The nurse adjusts the bed and asks if the client is comfortable. 3 The nurse leaves the door of the room open while attending to the client. 4 The nurse tells the client that the primary healthcare provider will visit soon.

2

The nurse asks questions to an older client about past experiences and listens attentively. Which therapeutic communication strategy is involved when the older client is recalling the past? 1 Touch 2 Reminiscence 3 Reality orientation 4 Validation therapy

2

The nurse is assessing a client after surgery. Which assessment finding does the nurse obtain from the primary source? 1 X-ray reports 2 Severity of pain 3 Results of blood work 4 Family caregiver interview

2

The nurse is assessing a client who had knee replacement surgery. Which assessment finding gathered by the nurse is an example of subjective data? 1 The client weighs 151 lbs (68.5 Kg). 2 The client's pain is 7 on a scale of 1 to 10. 3 The client's fasting blood sugar is 95 mg/dL. 4 The client's blood pressure is 140/90 mm/Hg.

2

The nurse is caring for a client before, during, and immediately after surgery. Which type of care is provided to the client? 1 Care that supports physical functioning 2 Care that supports homeostatic regulation 3 Care that supports psychosocial functioning 4 Care that provides immediate short-term help in physiological crises

2

The nurse is teaching the parent of an infant about inspecting the crib before putting an infant to sleep. Which statement made by the parent indicates a need for further education? 1 "I should remove mobiles from the infant." 2 "I should attach crib toys with hanging strings." 3 "I should check whether the crib's mattress fits snugly." 4 "I should disassemble and throw away the unsafe cribs."

2

The nurse noticed the breathing rate as regular and slow while assessing a client for respiration. What could be the condition of the client? 1 Apnea 2 Bradypnea 3 Tachypnea 4 Hyperpnea

2

The nurse pulls up on the client's skin and releases it to determine whether the skin returns immediately to its original position. What is the nurse assessing for? 1 Pain tolerance 2 Skin turgor 3 Ecchymosis formation 4 Tissue mass

2

The nurse tells a client undergoing diuretic therapy to avoid working in the garden on hot summer days. What condition is the nurse trying to prevent in this client? 1 Frostbite 2 Heatstroke 3 Hypothermia 4 Hyperthermia

2

What would be the respiratory rate in two-year-old child? 1 20 2 30 3 40 4 50

2

When providing care for a client with a nasogastric (NG) tube, the nurse should take measures to prevent what serious complication? 1 Skin breakdown 2 Aspiration pneumonia 3 Retention ileus 4 Profuse diarrhea

2

When suctioning a client with a tracheostomy, an important safety measure for the nurse is to do what? 1 Hyperventilate the client with room air before suctioning. 2 Apply suction only as the catheter is being withdrawn. 3 Insert the catheter until the cough reflex is stimulated. 4 Remove the inner cannula before inserting the suction catheter.

2

Which client assessment finding should the nurse document as subjective data? 1 Blood pressure 120/82 beats/min 2 Pain rating of 5 3 Potassium 4.0 mEq 4 Pulse oximetry reading of 96%

2

Which critical thinking skill refers to the use of knowledge and experience to choose effective client care strategies? 1 Evaluation 2 Explanation 3 Interpretation 4 Self-regulation

2

While assessing the muscle tone of a client, the client demonstrates a full range of muscle motion against gravity with some resistance. What score on the Lovett scale can be given to the client? 1 Fair (F) 2 Good (G) 3 Trace (T) 4 Normal (N)

2

While examining a client, a nurse finds a circumscribed elevation of the skin filled with serous fluid on the cheek. The lesion is 0.6 cm in diameter. What does the nurse suspect the finding to be? 1 Papule 2 Vesicle 3 Nodule 4 Pustule

2

The nurse observes that an older client seldom eats the meat on the meal trays. The nurse discusses this observation with the client, and the client states, "I only eat meat once a week because old people don't need protein every day." The nurse determines that the client needs to be taught about the: 1 Need for home-delivered meals 2 Foods that meet basic nutritional needs 3 Effect of aging on the need for some foods 4 Need for meat at least once per day throughout life

2 Foods that meet basic nutritional needs

our days after abdominal surgery a client has not passed flatus and there are no bowel sounds. Paralytic ileus is suspected. What does the nurse conclude is the most likely cause of the ileus? 1 Decreased blood supply 2 Impaired neural functioning 3 Perforation of the bowel wall 4 Obstruction of the bowel lumen

2 Impaired neural functioning Paralytic ileus occurs when neurological impulses are diminished as a result of anesthesia, infection, or surgery. Interference in blood supply will result in necrosis of the bowel. Perforation of the bowel will result in pain and peritonitis. Obstruction of the bowel initially will cause increased peristalsis and bowel sounds.

A nurse provides crutch-walking instructions to a client that has a left-leg cast. The nurse should explain that weight must be placed: 1 In the axillae. 2 On the hands. 3 On the right side. 4 On the side that the client prefers.

2 On the hands.

A nurse is caring for a client on bed rest. How can the nurse help prevent a pulmonary embolus? 1 Limit the client's fluid intake. 2 Teach the client how to exercise the legs. 3 Encourage use of the incentive spirometer. 4 Maintain the knee gatch position at an angle

2 Teach the client how to exercise the legs.

A nurse is caring for a client who has paraplegia as a result of a spinal cord injury. Which rehabilitation plan will be most effective for this client? 1 Arrangements will be made by the client and the client's family. 2 The plan is formulated and implemented early in the client's care. 3 The rehabilitation is minimal and short term because the client will return to former activities. 4 Arrangements will be made for long-term care because the client is no longer capable of self-care

2 The plan is formulated and implemented early in the client's care.

The nurse provides a client with left-sided weakness with instructions on how to safely use a cane. The nurse should demonstrate proper use of the cane by holding it on: 1 Alternating sides. 2 The right side. 3 The side of the weakness. 4 The side of the client's choice.

2 The right side.

A client is diagnosed with acquired immunodeficiency syndrome (AIDS). When examining the client's oral cavity, the nurse assesses white patchy plaques on the mucosa. The nurse recognizes that this finding most likely represents what opportunistic infection? 1 Cytomegalovirus 2 Histoplasmosis 3 Candida albicans 4 Human papillomavirus

3

A registered nurse (RN) is performing a physical examination of a client with chronic obstructive pulmonary disease. Which abnormal nail bed patterns can be expected in this client? 1 Spoon-shaped nails 2 Transverse depressions in nails 3 Softening of nail beds and flat nails 4 Red or brown linear streaks in nail bed

3

The nurse is assisting with the end-of-life care of an older adult. Which activity is performed when the nurse views family as context? 1 Assess the resources available to the family 2 Meet the client's family's comfort and nutritional needs 3 Meet the client's comfort, hygiene and nutritional needs 4 Determine the family's need for rest and their stage of coping

3

What would be the behavioral characteristic of a slow-to-warm up child according to the theory related to temperament? 1 Highly active 2 Irritable and irregular in habits 3 Negative reaction to new stimuli 4 A positive mild-to-moderately intense mood

3

When teaching about aging, the nurse explains that older adults usually have what characteristic? 1 Inflexible attitudes 2 Periods of confusion 3 Slower reaction times 4 Some senile dementia

3

The nurse is having difficulty understanding a client's decision to have hospice care rather than an extensive surgical procedure. Which ethical principle does the client's behavior illustrate? 1 Justice 2 Veracity 3 Autonomy 4 Beneficence

3 Autonomy The client is exhibiting the freedom to make a personal decision, and this reflects the concept of autonomy. Justice refers to fairness. Veracity refers to truthfulness. Beneficence refers to implementing actions that benefit others.

The nurse should instruct a client with an ileal conduit to empty the collection device frequently because a full urine collection bag may: 1 Force urine to back up into the kidneys. 2 Suppress production of urine. 3 Cause the device to pull away from the skin. 4 Tear the ileal conduit

3 Cause the device to pull away from the skin.

A nurse assesses the lungs of a client and auscultates soft, crackling, bubbling breath sounds that are more obvious on inspiration. This assessment should be documented as: 1 Vesicular 2 Bronchial 3 Crackles 4 Rhonchi

3 Crackles Crackles are abnormal breath sounds described as soft, crackling, bubbling sounds produced by air moving across fluid in the alveoli.

A nurse cares for a client that has been bitten by a large dog. A bite by a large dog can cause which type of trauma? 1 Abrasion 2 Fracture 3 Crush injury 4 Incisional laceration

3 Crush injury

A client reaches the point of acceptance during the stages of dying. What response should the nurse expect the client to exhibit? 1 Apathy 2 Euphoria 3 Detachment 4 Emotionalism

3 Detachment When an individual reaches the point of being intellectually and psychologically able to accept death, anxiety is reduced and the individual becomes detached from the environment. Although detached, the client is not apathetic but still may be concerned and use time constructively. Although resigned to death, the individual is not euphoric. In the stage of acceptance, the client is no longer angry or depressed.

Which nursing behavior is an intentional tort? 1 Miscounting gauze pads during a client's surgery. 2 Causing a burn when applying a wet dressing to a client's extremity. 3 Divulging private information about a client's health status to the media. 4 Failing to monitor a client's blood pressure before administering an antihypertensive.

3 Divulging private information about a client's health status to the media.

A nurse is preparing a community health program for senior citizens. The nurse teaches the group that the physical findings that are typical in older people include: 1 A loss of skin elasticity and a decrease in libido 2 Impaired fat digestion and increased salivary secretions 3 Increased blood pressure and decreased hormone production 4 An increase in body warmth and some swallowing difficulties

3 Increased blood pressure and decreased hormone production With aging, narrowing of the arteries causes some increase in the systolic and diastolic blood pressures; hormone production decreases after menopause. There may or may not be changes in libido; there is a loss of skin elasticity. Salivary secretions decrease, not increase, causing more difficulty with swallowing; there is some impairment of fat digestion. There may be a decrease in subcutaneous fat and decreasing body warmth; some swallowing difficulties occur because of decreased oral secretions.

A client is admitted with a diagnosis of premature labor. The nurse discovers that the client has been using heroin throughout her pregnancy. What is the most appropriate action for the nurse to take? 1 Notify the nurse manager of the unit. 2 Inform no one because all client information is confidential. 3 Inform the client's healthcare provider. 4 Alert the hospital security department because heroin is an illegal substance

3 Inform the client's healthcare provider. The fetus of a heroin-addicted mother is at risk for serious complications such as hypoxia and meconium aspiration. It is important to notify the healthcare provider of the client's heroin use, because this information will influence the care of the client and newborn.

A client with respiratory difficulties asks why the percussion procedure is being performed. The nurse explains that the primary purpose of percussion is to: 1 Relieve bronchial spasm. 2 Increase depth of respirations. 3 Loosen pulmonary secretions. 4 Expel carbon dioxide from the lungs.

3 Loosen pulmonary secretions. Percussion (chest physiotherapy) loosens pulmonary secretions by mechanical means. This is accomplished by vibrations over the lung fields on the client's posterior, anterior, and lateral chest. Percussion does not relieve bronchial spasms. Once pulmonary secretions are loosened by percussion and the client has a clearer airway, the depth of respirations may increase and facilitate removal of carbon dioxide from the lungs.

The nurse recognizes that the mental process most sensitive to deterioration with aging is: 1 Judgment 2 Intelligence 3 Creative thinking 4 Short-term memory

4 Short-term memory

The triage nurse in the emergency department receives four clients simultaneously. Which of the clients should the nurse determine can be treated last? 1 Multipara in active labor 2 Middle-aged woman with substernal chest pain 3 Older adult male with a partially amputated finger 4 Adolescent boy with an oxygen saturation of 91%

3 Older adult male with a partially amputated finger

The nurse discovers several palpable elevated masses on a client's arms. Which term most accurately describes the assessment findings? 1 Erosions 2 Macules 3 Papules 4 Vesicles

3 Papules Papules are superficial and elevated up to 0.5 cm. Nodules and tumors are masses similar to papules but are elevated more than 0.5 cm and may infiltrate deeper into tissues. Erosions are characterized as loss of the epidermis layer; macules are nonpalpable, flat changes in skin color less than 1 cm in diameter; and vesicles are usually transparent, filled with serous fluid, and are a blisterlike elevation.

A high-protein diet is recommended for a client recovering from a fracture. The nurse recalls that the rationale for a high-protein diet is to: 1 Promote gluconeogenesis. 2 Produce an anti-inflammatory effect. 3 Promote cell growth and bone union. 4 Decrease pain medication requirements

3 Promote cell growth and bone union. There is an increased need for protein with any type of body tissue trauma. High protein intake in the client with a fractured bone promotes cell growth and therefore bone union. High protein intake during recovery from a bone fracture is not related to gluconeogenesis, inflammation, or pain.

Which drug requires the nurse to monitor the client for signs of hyperkalemia? 1 Furosemide (Lasix) 2 Metolazone (Zaroxolyn) 3 Spironolactone (Aldactone) 4 Hydrochlorothiazide (HydroDIURIL

3 Spironolactone (Aldactone) Spironolactone is a potassium-sparing diuretic; hyperkalemia is an adverse effect. Furosemide, metolazone, and hydrochlorothiazide generally cause hypokalemia.

A nurse is caring for a client diagnosed with Methicillin-Resistant Staphylococcus Aureus (MRSA) in the urine. The health care provider orders an indwelling urinary catheter to be inserted. Which precaution should the nurse take during this procedure? 1 Droplet precautions 2 Reverse isolation 3 Surgical asepsis 4 Medical asepsis

3 Surgical asepsis

While performing a physical assessment of a female client, the nurse positions the client in Sims' position. Which body system will be assessed in this position? Select all that apply. 1 Heart 2 Vagina 3 Rectum 4 Female genitalia 5 Musculoskeletal system

3,4,5

A client complains of pain in the ear. While examining the client, a nurse finds swelling in front of the left ear. Which lymph node does the nurse expect to be involved? 1 Mastoid 2 Occipital 3 Submental 4 Pre-auricular

4

A client has a history of a persistent cough, hemoptysis, unexplained weight loss, fatigue, night sweats, and fever. Which risk should be assessed? 1 Lung cancer 2 Cerebrovascular disease 3 Cardiopulmonary alterations 4 Human immunodeficiency virus (HIV) infection

4

A client has relocated to a new city for work. The client is unable to continue the practice of walking for 30 minutes daily and exercising five days a week. Which stage of the transtheoretical model of health behavior change is the client experiencing? 1 Action 2 Preparation 3 Maintenance 4 Precontemplation

4

A nurse assesses a client with dry and brittle hair, flaky skin, a beefy-red tongue, and bleeding gums. The nurse recognizes that these clinical manifestations are most likely a result of what? 1 A food allergy 2 Noncompliance with medications 3 Side effects from medications 4 A nutritional deficiency

4

A nurse in the health clinic is counseling a college student who recently was diagnosed with asthma. On what aspect of care should the nurse focus? 1 Teaching how to make a room allergy-free 2 Referring to a support group for individuals with asthma 3 Arranging with the college to ensure a speedy return to classes 4 Evaluating whether the necessary lifestyle changes are understood

4

A nurse is assessing a client's nails and finds a slight convex curve at the angle from the skin to nail base of about 160 degrees. Which condition does the nurse suspect? 1 Clubbing 2 Paronychia 3 Koilonychia 4 Normal finding

4

A nurse is assessing several clients. Which client will require parenteral nutrition? 1 A client with brain neoplasm 2 A client with anorexia nervosa 3 A client with inflammatory bowel disease 4 A client with severe malabsorption disorder

4

A nurse is reviewing a client's plan of care. What is the determining factor in the revision of the plan? 1 Time available for care 2 Validity of the problem 3 Method for providing care 4 Effectiveness of the interventions

4

A nurse suspects that a client has interacted with poison ivy. Assessment findings reveal vesicles on the arms and legs. Which is the description of a vesicle? 1 A lesion filled with purulent drainage 2 An erosion into the dermis 3 A solid mass of fibrous tissue 4 A lesion filled with serous fluid

4

An 82-year-old retired schoolteacher is admitted to a nursing home. During the physical assessment, the nurse may identify which ocular problem common to persons at this client's developmental level?: 1 Tropia 2 Myopia 3 Hyperopia 4 Presbyopia

4

An obese adult develops an abscess after abdominal surgery. The wound is healing by secondary intention and requires repacking and redressing every 4 hours. Which diet should the nurse expect the healthcare provider to prescribe to best meet this client's immediate nutritional needs? 1 Low in fat and vitamin D 2 High in calories and fiber 3 Low in residue and bland 4 High in protein and vitamin C

4

How does the World Health Organization (WHO) define "health"? 1 A condition when people are free of disease 2 A condition of life rather than pathological state 3 An actualization of inherent and acquired human potential 4 A state of complete physical, mental, and social well-being

4

On the third postoperative day after a below-the-knee amputation, a client is refusing to eat, talk, or perform any rehabilitative activities. What is the best initial approach that the nurse should take when interacting with this client? 1 Explain why there is a need to increase activity. 2 Emphasize that with a prosthesis, there will be a return to the previous lifestyle. 3 Appear cheerful and noncritical regardless of the client's response to attempts at intervention. 4 Acknowledge that the client's withdrawal is an expected and necessary part of initial grieving.

4

Refusing to follow the prescribed treatment regimen, a client plans to leave the hospital against medical advice. What is it important for the nurse to inform the client of? 1 That the client is acting irresponsibly 2 That this action violates the hospital policy 3 That the client must obtain a new primary healthcare provider for future medical needs 4 That the client must accept full responsibility for possible undesirable outcomes

4

The home healthcare nurse visits a client who lives with her two grandchildren. The client's daughter is a single-parent who is away at work and comes home only on weekends. Which term does the nurse use to define this family form? 1 Nuclear family 2 Extended family 3 Single-parent family 4 Skip-generation family

4

The nurse is caring for a surgical client who develops a wound infection during hospitalization. How is this type of infection classified? 1 Primary 2 Secondary 3 Superinfection 4 Nosocomial

4

The nurse is developing a plan of care for the client who has activity intolerance. In determining the desired client outcomes, what should the nurse do? 1 Prioritize psychosocial needs over physical needs. 2 Use the Nursing Outcomes Classification (NOC) only. 3 Use nursing knowledge to plan outcomes and disregard client and family desires. 4 Set priorities and outcomes using the client's and family input.

4

The nurse is performing a weight assessment for different people in a community. Which question should the nurse ask a client to determine a disease-related change in weight? 1 Do you follow a strict calorie intake? 2 Have you notices any changes in the social aspects of eating? 3 Are you taking diuretics or insulin? 4 Have you noticed any unintentional weight loss in the past six months?

4

The nurse is preparing to assess the four abdominal quadrants of a client who complains of stomach pain. When determining the order of the assessment, the nurse recognizes that it is important to assess the symptomatic quadrant when? 1 First 2 Second 3 Third 4 Last

4

The nurse is transferring a client from the bed to the chair. Which action should the nurse take during the transfer? 1 Place the client in a semi-Fowler position. 2 Stand behind the client during the transfer. 3 Turn the chair so it faces away from the bed. 4 Instruct the client to dangle the legs.

4

The nurse receives information about a client through another nurse. The nurse then finds that information has some missing facts. Which critical thinking attitude would the nurse use to clarify the information after talking to the client directly? 1 Fairness 2 Humility 3 Discipline 4 Perseverance

4

The nurse recognizes that which is the mental process most sensitive to deterioration with aging? 1 Judgment 2 Intelligence 3 Creative thinking 4 Short-term memory

4

The primary healthcare provider treats a client with a pressure ulcer. While assessing the client, the nurse identifies exposed bone and tendons. Which stage does the nurse document for this pressure ulcer?

4

What does a nurse consider the most significant influence on many clients' perception of pain when interpreting findings from a pain assessment? 1 Age and sex 2 Physical and physiological status 3 Intelligence and economic status Correct4 Previous experience and cultural values

4

Which caring intervention helps to provide comfort, dignity, respect, and peace to a client? 1 Listening 2 Spiritual caring 3 Providing presence 4 Relieving pain and suffering

4

Which critical thinking skill does the nurse associate with the concept of maturity? 1 Eagerness to acquire knowledge 2 Being tolerant of different views 3 Trust in own reasoning processes 4 Ability to reflect on own judgments

4

Which nursing action indicates that the nurse is actively listening to the client? 1 The nurse states his or her own opinions when the client is speaking. 2 The nurse refrains from telling his or her own story to the client. 3 The nurse reads the client's health record during the conversation. 4 The nurse interprets what the client is saying and reiterates in his or her own words.

4

Which of the following is a description of the percussion technique? 1 Listening to sounds that the body makes 2 Using the sense of touch to assess and collect data 3 Carefully looking for abnormal findings 4 Tapping the skin with the fingertips to vibrate underlying tissues

4

Which physical assessment of the skin indicates that a client is addicted to phencyclidine? 1 Burns 2 Vasculitis 3 Diaphoresis 4 Red and dry skin

4

Which positioning should be avoided while assessing a client with a history of asthma? 1 Sitting 2 Supine 3 Dorsal recumbent 4 Lateral recumbent

4

Which statement is true about the nursing model "team nursing"? 1 The registered nurse is responsible for all aspects of client care. 2 Client care can be delegated to other healthcare team members. 3 The registered nurse works directly with the client, family members, and healthcare team members. 4 Hierarchical communication exists from charge nurse to charge nurse, charge nurse to team leader, and team leader to team members.

4

Which statement is true for attachment in the newborn? 1 Attachment occurs for the first 28 days. 2 Attachment begins in the first week of birth. 3 Attachment is the overlapping of soft skull bones. 4 Attachment is the interaction between parent and child.

4

Which statement is true for collaborative problems in a client receiving healthcare? 1 They are the identification of a disease condition. 2 They include problems treated primarily by nurses. 3 They are identified by the primary healthcare provider. 4 They are identified by the nurse during the nursing diagnosis stage.

4

Which theory proposes that older adults experience a shift from a materialistic to cosmic view of the world? 1 Activity theory 2 Continuity theory 3 Disengagement theory 4 Gerotranscendence theory

4

While assessing a client for the dorsalis pedis pulse, a nurse documents the reading as 1+. What can be inferred from this finding? 1 There is absence of a pulse. 2 The pulse strength is normal. 3 The pulse strength is bounding. 4 The pulse strength is barely palpable.

4

While caring for a client with a Hemovac portable wound drainage system, the nurse observes that the collection container is half full. The nurse empties the container. What is the next nursing intervention? 1 Encircle the drainage on the dressing. 2 Irrigate the suction tube with sterile saline. 3 Clean the drainage port with an alcohol wipe. 4 Compress the container before closing the port.

4

A recent immigrant from mainland China is critically ill and dying. What question should the nurse ask when collecting information to meet the emotional needs of this client? 1 "Do you like living in this country?" 2 "When did you come to this country?" 3 "Is there a family member who can translate for you?" 4 "Which family member do you prefer to receive information?"

4 "Which family member do you prefer to receive information?"

hen planning discharge teaching for a young adult, the nurse should include the potential health problems common in this age group. What should the nurse include in this teaching plan? 1 Kidney dysfunction 2 Cardiovascular diseases 3 Eye problems, such as glaucoma 4 Accidents, including their prevention

4 Accidents, including their prevention

Which of the following legal defenses is the most important for a nurse to develop? 1 Dedication 2 Certification 3 Assertiveness 4 Accountability

4 Accountability The concept of accountability is of high priority in nursing practice. As a licensed professional, the nurse is always accountable, which means liable and answerable for his or her actions.

A nurse in the surgical intensive care unit is caring for a client with a large surgical incision. The nurse reviews a list of vitamins and expects that which medication will be prescribed because of its major role in wound healing? 1 Vitamin A (Aquasol A) 2 Cyanocobalamin (Cobex) 3 Phytonadione (Mephyton) 4 Ascorbic acid (Ascorbicap)

4 Ascorbic acid (Ascorbicap) Vitamin C (ascorbic acid) plays a major role in wound healing. It is necessary for the maintenance and formation of collagen, the major protein of most connective tissues. Vitamin A is important for the healing process; however, vitamin C is the priority because it cements the ground substance of supportive tissue. Cyanocobalamin is a vitamin B12 preparation needed for red blood cell synthesis and a healthy nervous system. Phytonadione is vitamin K, which plays a major role in blood coagulation.

A terminally ill client is furious with one of the staff nurses. The client refuses the nurse's care and insists on doing self-care. A different nurse is assigned to care for the client. What should be the newly assigned nurse's initial step in revising the client's plan of care? 1 Get a full report from the first nurse and adjust the plan accordingly. 2 Ask the health care provider for a report on the client's condition and plan appropriately. 3 Tell the client about the change in staff responsibilities and assess the client's reaction. 4 Assess the client's present status and include the client in a discussion of revisions to the plan of care

4 Assess the client's present status and include the client in a discussion of revisions to the plan of care

A nurse working in an emergency department is concerned about a recent increase in malpractice claims against nurses. What is the best way for the nurse to avoid being named in a lawsuit? 1 Carry malpractice insurance. 2 Write vague incident reports. 3 Transfer to another department. 4 Attend professional development programs.

4 Attend professional development programs. The best ways to prevent professional negligence (malpractice) are to attend continuing education programs and improve practice; additional education is advisable when one is working in specialty areas, such as emergency departments or intensive care areas.

The unlicensed assistive person (UAP) assigned to the 7 am shift has not been coming to work until 8 am. Nursing care is delayed and assignments are started late. What is the most appropriate action by the charge nurse/team leader? 1 Discuss the issue with a friend from another unit 2 Remind the UAP of the expected start time 3 Report the problem to the Human Resources department 4 Document the information before discussing it with the UAP

4 Document the information before discussing it with the UAP

A nurse in the health clinic is counseling a college student who recently was diagnosed with asthma. On what aspect of care should the nurse focus? 1 Teaching how to make a room allergy-free. 2 Referring to a support group for individuals with asthma. 3 Arranging with the college to ensure a speedy return to classes. 4 Evaluating whether the necessary lifestyle changes are understood

4 Evaluating whether the necessary lifestyle changes are understood

The nurse is teaching a client about adequate hand hygiene. What component of hand washing should the nurse include that is most important for removing microorganisms? 1 Soap 2 Time 3 Water 4 Friction

4 Friction

Which action by a home care nurse would be considered an act of euthanasia? 1 Implementing a "do not resuscitate" order in the home health setting. 2 Abiding by the decision of a living will signed by the client's family. 3 Encouraging a client to consult an attorney to document and assign a power of attorney. 4 Knowing that a dying client is overmedicating and not acting on this information.

4 Knowing that a dying client is overmedicating and not acting on this information. In this situation being aware that a client is overmedicating and taking no action can be considered an act of euthanasia on the part of the home care nurse. Implementing a "do not resuscitate" order, abiding by the decision of a living will signed by the client's family, and encouraging the client to consult an attorney are all appropriate actions for a home care nurse.

A client is admitted voluntarily to a psychiatric unit. Later, the client develops severe pain in the right lower quadrant and is diagnosed as having acute appendicitis. How should the nurse prepare the client for the appendectomy? 1 Have two nurses witness the client signing the operative consent form. 2 Ensure that the health care provider and the psychiatrist sign for the surgery because it is an emergency procedure. 3 Ask the client to sign the operative consent form after the client has been informed of the procedure and required care. 4 Inform the client's next of kin that it will be necessary for one of them to sign the consent form because the client is on a psychiatric unit.

3 Ask the client to sign the operative consent form after the client has been informed of the procedure and required care.

Which nursing activities are examples of primary prevention? Select all that apply. 1 Preventing disabilities 2 Correcting dietary deficiencies 3 Establishing goals for rehabilitation 4 Assisting with immunization programs 5 Facilitating a program about smoking cessation

4 Assisting with immunization programs 5 Facilitating a program about smoking cessation Immunization programs prevent the occurrence of disease and are considered primary interventions. Stopping smoking prevents the occurrence of disease and is considered a primary intervention. Preventing disabilities is a tertiary intervention. Correcting dietary deficiencies is a secondary intervention. Establishing goals for rehabilitation is a tertiary intervention. Topics

A nurse prioritizes client care using Maslow's hierarchy of needs. Which situation should the nurse address first according to Maslow's hierarchy?

A client complains of sleeplessness due to pain post-surgery.

The client's serum sodium is 123 mEq/L (123 mmol/L). Which prescription should the nurse question?

Administer intravenous saline (NS) at 125 mL/hr

The registered nurse is teaching a student nurse about nursing roles while caring for a client with skin conditions. Which task could be delegated to a licensed practical nurse in this situation?

Applying dressings and administering medications

The nurse is caring for a client admitted with chronic obstructive pulmonary disease (COPD). The nurse should monitor the results of which laboratory test to evaluate the client for hypoxia? 1 Red blood cell count 2 Sputum culture 3 Arterial blood gas 4 Total hemoglobin

Arterial blood gas All of these laboratory tests assist in the evaluation of a client with respiratory difficulties; however, arterial blood gas analysis is the only test that evaluates gas exchange in the lungs. This provides accurate information about the client's oxygenation status.

Which of these cultural groups adopts a combination of dietary, herbal, and other naturalistic therapies to prevent and treat illness?

Asian Indian

Which intervention by the nurse is an important aspect of client-centered care according to the survey conducted by the Picker Institute?

Asking the client if the family should be involved in his or her care

What should be the priority action of the nurse who is caring for a client with a leg in traction?

Assessing skin integrity

What is the correct order of phases of the nursing process?

Assessment Diagnosis Planning Implementation Evaluation

A daughter of a Chinese-speaking client approaches a nurse and asks multiple questions while maintaining direct eye contact. What culturally related concept does the daughter's behavior reflect? 1 Prejudice 2 Stereotyping 3 Assimilation 4 Ethnocentrism

Assimilation Assimilation involves incorporating the behaviors of a dominant culture. Maintaining eye contact is characteristic of the American culture and not of Asian cultures. Prejudice is a negative belief about another person or group and does not characterize this behavior. Stereotyping is the perception that all members of a group are alike. Ethnocentrism is the perception that one's beliefs are better than those of others.

Which would the nurse consider to be a potential respiratory system-related complication of surgery?

Atelectasis

A nurse is caring for a client with quadriplegia. Which nursing intervention will decrease the occurrence of pressure ulcers?

A nurse is caring for a client with quadriplegia. Which nursing intervention will decrease the occurrence of pressure ulcers?

A nursing student is taking down notes about paradigm. Which point noted down by the nursing student needs correction?

A paradigm is the perspective of a profession

The nurse is assessing a client with arthritis. Which statement made by the client indicates a precipitating factor that is an intellectual standard for critical thinking?

"I run for 30 minutes every day; this exercise increases my pain.

The nurse teaches a client about cleaning the skin to prevent pressure ulcers. Which statement made by the client indicates the nurse needs to follow up?

"I should apply powders or talc on a perineum wound."

The primary healthcare provider prescribes an adrenergic agonist to a client with increased intraocular pressure. Which question is priority that the nurse should ask the client?

"Do you take antidepressants?"

A nurse is teaching an older adult client about managing chronic pain with acetaminophen. Which client statement indicates that the teaching is effective?

"I have to be careful about which over-the-counter cold preparations I take when I have a cold."

Which statement is true about Betty Neuman's theory?

Betty Neuman's theory is based on stress and the client's reaction to the stressor.


Kaugnay na mga set ng pag-aaral

NUR 236 PrepU Chapter 28: Growth and Development of the School-Age Child

View Set

Evidence Based Medicine - Spring 2015

View Set

Chapter 15: Doctor Patient Communication

View Set

ATI Questions for Meds, Older Adult, Sensory, and CAT

View Set

Network+Guide Chapter-6 Review Question

View Set

decimal, percentage and fractions

View Set

Chapter 58: Caring for Clients with Disorders of the Kidneys and Ureters

View Set